[Burichan] [Futaba] [Gurochan] [Photon] [Tomorrow] - [Главная] [Управление]

[Назад]
Ответ
Leave these fields empty (spam trap):
Имя
E-mail
Тема
Сообщение
Файл
Подтверждение
Пароль (для удаления файлов и сообщений)
  • Supported file types are: GIF, JPG, PNG
  • Maximum file size allowed is 1000 KB.
  • Images greater than 200x200 pixels will be thumbnailed.

RussellSet.jpg (0.0 KB, -1x-1)
0 No.160159  
Наша кафедра продолжает свою работу. Здесь мы обсуждаем математику, а также иногда отвечаем на вопросы доброанонов.

Тред обучения математике: >>124265
Репетиторство и школьная математика обсуждается тоже в нём.

Библиотека: http://gen.lib.rus.ec
Скачать статью бесплатно без смс: https://sci-hub.io
Ответы на вопросы: http://mathoverflow.net
Если хочется что-нибудь порешать: http://ium.mccme.ru/old_courses.html

Калькуляторы:
http://online-integral.ru
http://www.wolframalpha.com

Предыдущий: >>157896
>> No.160167  
1496956160030.png (0.0 KB, -1x-1)
0
>> No.160178  
1497108514560.png (0.0 KB, -1x-1)
0
Биекция - этой свойство функции (отображения). Одна и также функция ведь может быть при одних образе/прообразе биекцией, а при других уже не быть? То есть получается, что это свойство не только функции, но и совокупности образа/прообраза? Или если мы заменяем множества, то и функция уже формально иная?
>> No.160179  
>>160178
Зависит от того, как определена функция. Обычно функцию определяют как тройку (X, ф, Y), где ф - подмножество декартова произведения XY такое, что для любого x из X существует единственный y из Y такой, что xфy.
>> No.160180  
>>160179
Угу, то есть если мы условие выбора подмножества ф и образ Х не меняем, и поменяем Y, то это будет уже другая функция, и если биекция пропадет, то ничего удивительного. Спасибо.
>> No.160187  
Вопрос от химика, знающего математику мало(в сравнении с самими математиками). Во всех вводных лекциях по физической химии слышал фразу, что уравнение Шрёдингера для атомов с несколькими электронами нерешаемо. Почему? В силу каких-то фундаментальных аксиом или же оно требует для решения невероятных даже с современными технологиями количеств расчётов?
>> No.160188  
1497069491203.jpg (0.0 KB, -1x-1)
0
>>160187
Оно аналитически нерешаемо также как задача трех тел, если частиц больше 2. То есть в случае гелия уже провал. Численно можно решать, но в реальных системах слишком много частиц, поэтому в лоб тоже не выйдет. Погугли теория функционала плотности. С помощью таких методов находят решения численно. Довольно затратно по компьютерным ресурсам, но в принципе решают. Еще я видел инфу, что задачу многочастичной системы (как задачу трех тел, только тел больше) именно в квантовомеханическом случае решил Фадеев. https://ru.wikipedia.org/wiki/%D0%A3%D1%80%D0%B0%D0%B2%D0%BD%D0%B5%D0%BD%D0%B8%D1%8F_%D0%A4%D0%B0%D0%B4%D0%B4%D0%B5%D0%B5%D0%B2%D0%B0
>> No.160243  
14960903002430.jpg (0.0 KB, -1x-1)
0
Как найти базис пересечения классов Поста?
Например, у меня есть задание: привести примеры базисов в М ∩ Т1, в Т0 ∩ М, в L ∩ М.
В голову приходит только взять из известных базисов функции, которые принадлежат обоим классам и объединить их. То что ничего лишнего из них не выразится — это понятно, но как потом доказывать, что выражаются все функции пересечения? А потом еще проверять, не выражаются ли они через друг-друга довольно затратно по времени. И вообще, это объединение получается, похоже.
>> No.160276  
>>160243
Никакого общего метода нет(насколько я знаю), но он и не нужен. В твоих задачах надо подумать и понять, что пересечения классов получаются маленькие. Например, L ∩ М состоит только из х и констант: если линейная функция зависит от двух и более переменных, то подстановкой нулей вместо всех переменных, кроме двух, из неё можно получить х+у либо х+у+1, а это немонотонные функции, т.е. линейная функция бывает монотонной только когда она зависит только от одной переменной.
>> No.160284  
>>160276
Спасибо.
>> No.160289  
Итак, есть следующие вещи.
(1).Полукольцо, (2)кольцо, (3)сигма кольцо, (4)дзета кольцо, (5)алгебра, (6)сигма алгебра.
Не могу придумать примеры
4, но не 3
2, но не 3 или 4
5, но не 6
>> No.160291  
>>160283
> дзета кольцо
Гугл находит только «пылепоглощающие кольца ZETA».
>> No.160295  
>>160289
> сигма кольцо
> дзета кольцо

Определения в студию.
>> No.160296  
>>160289
Я(как, подозреваю, и большинство посетителей этого треда)понятия не имею, что такое "дзета-кольцо". Думаю, это доморощенный термин, придуманный автором твоего курса по алгебре. Так что от тебя требуются некоторые пояснения.

> 5, но не 6

Возьмем бесконечномерное линейной пространство, состоящие из бесконечных последовательностей, в которых только конечное число элементов ненулевое. Операции сложения самые обычные, покоординатные(иными словами, мы берем прямую сумму(не путать с прямым произведением)бесконечного числа экземпляров поля). Это, очевидно, алгебра. Сигма-алгеброй оно не является, т.к. сумма бесконечного числа элементов этой алгебры в ней может и не лежать(например, возьмем элементы (1, 0, 0, ....., 0, ...), (0,1, 0, ..., 0, ), (0, 0, 1, 0, ...., 0, ...), .....).
>> No.160297  
>>160295
>>160291
Я вообще уже нашел все примеры в задачнике Ульянова Бахвалова.
Сигма кольцо - кольцо, замкнутое относительно не только пересечения и симм. разности, но и относительно счетного объединения. Дзета кольцо - относительно счетного пересечения.
>> No.160300  
>>160297
> Дзета кольцо - относительно счетного пересечения.
Это же дельта -кольцо.
>> No.160308  
putin-2.jpg (0.0 KB, -1x-1)
0
>>160300
Я оставлю этот момент без комментариев и просто закрою вкладку, потупив взор.
>> No.160336  
Анон, что такое автоморфные функции в двух словах?
>> No.160337  
>>160336
В двух словах - голоморфные функции, имеющие нетривиальную группу автоморфизмов. Частным(и, видимо, наиболее изученным) случаем являются модулярные функции веса нуль.

https://en.m.wikipedia.org/wiki/Modular_form

Изучение этих функций интересно потому что при факторизации алгебраических многообразий по группам автоморфизмов могут получиться новые алгебраические многообразия. Например, фактор проективной прямой по группе сдвигов - кривая третьей степени. Поэтому изучение автоморфных функций относительно группы сдвигов равносильно изучению кривых третьей степени. С другой стороны, фактор по группе может и не быть многообразием(хрестоматийный пример - модулярная группа). Изучение таких случаев интересно потому, что в алгебраичнской геометрии на сегодняшний день нет разработанной теории для работы с факторами, не являющимися многообразиями, и изучение автоморфных функций может дать ключи к созданию такой теории.
>> No.160340  
>>160337
Спасибо за ответ. Но я слишком тупой, чтобы понять что все это значит. И почему я подумал, что если спросить у кого-то, то все сразу станет ясно? Извини за вопрос, на который я не ожидал ответа.
>> No.160343  
>>160340
Что именно ты не понял? Если ты не понял "всё сразу", то это значит, что автоморфные функции тебе пока не нужны просто. Бессмысленно разбирать автоморфные функции, не имея представления о голоморфных функция, группах автоморфизмов и комплексных многообразиях.
>> No.160345  
>>160343
Попробуй не отвечать в стиле >>160155 и показать несколько хороших, годных примеров.
>> No.160346  
>>160345
Да нет, он прав, они мне не нужны. Просто я пытаюсь в эту алгебру въехать уже года два с перерывами, и получается хреново.
>> No.160348  
>>160346
Большинство учебников по алгебре совершенно чудовищны, можно их годами читать и так и не въехать.
Сам могу посоветовать только читать Алюффи.
>> No.160349  
>>160348
> въехать в алгебру
> Алюффи

Ты там совсем со своим теоркатом поехал? От этой хрени только еще больше запутываешься.
>> No.160354  
>>160345
Примеров чего? Автоморфных функций?
Я назвал два примера: модулярные формы веса нуль(автоморфные функции относительно модулярной группы), и автоморфные функции относительно группы сдвигов, т. е. функции на эллиптической кривой. Конечно, на второй пример я ссылок не привел. Ну, допустим, вот ссылка: https://en.m.wikipedia.org/wiki/Weierstrass%27sellipticfunctions

Но если ты не можешь самостоятельно вырубиться в написанное по ссылкам, то, увы, я не могу тебе с нуля объяснить их содержание в рамках поста на АиБ, даже если б хотел. Не вся математика элементарна. Ключевые слова по темам, которые нужно освоить, чтобы разбирать этот вопрос, я назвал. Эллиптические функции разбираются в любом хорошем учебнике комплексного анализа, например, у Шабата. Ищите и обрящете.
>> No.160355  
>>160348
> можно их годами читать и так и не въехать.

Можно, если не понимать, зачем ты это делаешь.
>> No.160357  
universal-quotient1.png (0.0 KB, -1x-1)
0
>>160349
> От этой хрени только еще больше запутываешься.
Только если ты — типичный алгебраист. Нормальные люди скорее поймут пикрелейтед чем >>160155 или (уж тем более) объянение фактор-групп в большинстве учебников алгебры.

>>160354
Почему топологи умеют в примеры, а алгебраисты — не умеют? Открываешь книгу книгу по дифференциальной топологии — куча картинок, всё понятно. Открывешь какого-нибудь Ван дер Вардена — картинок нет, коммутативных диаграмм нет, ничего не понятно. Причём, Ван дер Варден же лучше большинства других учебников.
>> No.160358  
_group_shit.png (0.0 KB, -1x-1)
0
>>160357
> картинок нет
Нарисуй GF(81).
> коммутативных диаграмм нет
Не нужны.
> ничего не понятно
Просто твой мозк пожрал теоркат.

Ладно, я немножк тралирую. Но я не понимаю пользы от теорката. Вот смотри, группа с точки зрения нормального человека - это множество M и операция ∙: M×M -> M, которая удовлетворяет трем простым критериям ∀a,b,c:(ab)c=a(bc); ∃e:xe=ex=x∀x; ∀x∃y:xy=yx=e. С точки зрения Алуффи, это какое-то абстрактное пикрилетед говно, в котором ничего не понятно. Нахуй оно мне нужно?
>> No.160359  
0JgCDWXtRjU.jpg (0.0 KB, -1x-1)
0
>>160358
> С точки зрения Алуффи, это какое-то абстрактное пикрилетед говно
Пикрелейтед — не группа, а категория* групп. Каждый квадрат всего навсего определение гомоморфмзма. Суть очень проста:
- Пусть есть группы (G, +), (H, ·), (K, ×)
- φ: G → H, причём для любых g1, g2: φ(g1+g2) = φ(g1)·φ(g2)
- ψ: H → K, причём для любых h1, h2: ψ(h1·h2) = ψ(h1)×ψ(h2)
- (φ;ψ)(g) := ψ(φ(g)), причём для любых g1, g2: (φ;ψ)(g1+g2) = (φ;ψ)(g1)×(φ;ψ)(g2)
В трёх словах: композиция гомоморфизмов — гомоморфизм.
А группа в теоркате определяется очень просто: группоид с единственным элементом. Точнее, его морфизмы.
> Но я не понимаю пользы от теорката.
1. Диаграммы позволяют выразить алгебраические конструкции визуально.* Вместо трудночитаемого супа из символов — двухмерный или трёхмерный каркас из стрелочек. Правда иногда эту визуальность портит дурацкая нотация вроде g∘f вместо f;g (diagrammatic order) для композиции, но ведь всегда же можно использовать свою нотацию вместо стандартной.
2. Мостик для всего. Универсальные свойства не просто так называются, они везде одни и те же. А вот соответствующие сущности — разные: фактор-объект становится фактор-множеством в категории множеств и фактор-группой в категории групп, копроизведение это дизъюнктное объединение для множеств и прямая сумма для абелевых групп. Аналогии — основа математической интуиции, а теоркат — язык аналогий.
> Нарисуй GF(81).
Это же циклическая группа? Совсем неинтересно.
Но ты, вероятно, хочешь ехидно спросить: как же визуализировать группы? Есть два очень мощных инструмента: граф Кэли и граф циклов. Само собой, они не показывают всех* отнощений в группе, но тем не менее позволяют визуально рассуждать как о конкретных группах, так и о понятиях теории групп.
Пикрелейты — из Visual Group Theory (есть как книга, так и лекции). Эта книга, как и Алюффи — лучшее, что случалось с учебниками по теории групп за всё время их существования. Она идеально дополняет Algebra: Chapter 0: если читая Алюффи, ты видишь абстрактное, то читая VGT, видишь конкретное. Но видишь, а не просто глотаешь невкусную кашу из символов.
Кроме книг есть ещё и эксплорер: http://groupexplorer.sourceforge.net/ge2intro.html
>> No.160360  
>>160359
> Это же циклическая группа?
Погуглив, написал ерудну. Надо было читать, что читал.
Я совсем не заглядывал в поля, в теорию групп учил лишь чтобы понимать алгебраическую топологию. Так что ответить не смогу.
>> No.160363  
>>160359
> Вместо нормальных человеческих символов и слов — трудночитаемый двух- или вообще трехмерный суп из стрелочек.
Пофиксил.
> - φ: G → H, причём для любых g1, g2: φ(g1+g2) = φ(g1)·φ(g2)
> - ψ: H → K, причём для любых h1, h2: ψ(h1·h2) = ψ(h1)×ψ(h2)

φ,ψ линейны. короче, не правда ли?
> - (φ;ψ)(g) := ψ(φ(g)), причём для любых g1, g2: (φ;ψ)(g1+g2) = (φ;ψ)(g1)×(φ;ψ)(g2)
Их композиция тоже линейна (спасибо, кэп)

Выходит, что я открыл Алуффи там где написано Group Definition, а там вместо ожидаемого определения групп черт знает что. И ты еще ругаешь Ван дер Вардена! У него-то если уж написано "определение X", то за этим словом идет определение X.

> 1. Диаграммы позволяют выразить алгебраические конструкции визуально.
Это ваше "визуально" - пустое нагромождение стрелок.
> 2. Мостик для всего.
Как раз спрашивал на эту тему в прошлом треде и вот ответ:
>>159237
С одной стороны, "мостик для всего", а как припрешь, так сразу "Что это?" и "Нужен контекст". Выходит, совсем эта ваша каша из стрелочек не универсальна. И как правило, куда более громоздка, чем то же самое, написанное словами.

>>160360
Вот именно, даже простейшее конечное поле ты уже не нарисуешь так, чтобы рисунок имел хоть какую-то ценность. Так что довольно странно придираться, что в книгах о трансформации фигур есть картинки, а в книгах по алгебре нет.
>> No.160364  
>>160363
> Выходит, совсем эта ваша каша из стрелочек не универсальна. И как правило, куда более громоздка, чем то же самое, написанное словами.
Книжка - универсальный способ передачи знаний на расстоянии и во времени, однако, не всякая книжка по дефолту окажется тебе полезной. Для чтения иных тебе придётся освоить громоздкую грамматику чужого языка и его лексику, причём, лексика традиционно многозначна и в специфической области используется своё множество терминов в виде перегруженных смыслом слов общего словаря. Это тоже вызывает у тебя негодование?
Не тот анон, не поборник теорката с первого класса, а просто jftgj
>> No.160365  
>>160363
Няша, тебе уже сказали, как определяется группа. Группой называется категория с одним объектом, в которой каждый морфизм - изоморфизм. К этому определению претензии есть?
>> No.160366  
group.png (0.0 KB, -1x-1)
0
>>160363
> Выходит, что я открыл Алуффи там где написано Group Definition, а там вместо ожидаемого определения групп черт знает что
У нас с тобой, похоже, разный Алюффи.
> φ,ψ линейны
Можно даже проще: φ,ψ — гомоморфизмы.
> Это ваше "визуально" - пустое нагромождение стрелок.
> Вместо нормальных человеческих символов и слов — трудночитаемый двух- или вообще трехмерный суп из стрелочек.

А вот это сильно зависит от восприятия. Некоторые люди могут программировать вот так: https://www.youtube.com/watch?v=iWXebEeGwn0
Я так не могу. Я весьма полагаюсь на чёткое и цветное зрение как в жизни, так и в размышлениях. И потому хочу видеть.*
Алгебраистам, судя по всему, зрение не нужно. Если прогнать Ван дер Вардена через экранный диктор, ничего, в принципе, не изменится.
> С одной стороны, "мостик для всего", а как припрешь, так сразу "Что это?" и "Нужен контекст".
https://en.wikipedia.org/wiki/Universal_property
Это не совсем обычные диаграммы.
> Вот именно, даже простейшее конечное поле ты уже не нарисуешь так, чтобы рисунок имел хоть какую-то ценность.
Опять же, не знаю. Может, там есть какая-то красивая симметрия, позволяющая сделать наглядный трёхмерный граф с 81 узлом (это, в принципе, не так уж и много). Может, часть графа можно заменить на многоточия?
Не знаю.
> Так что довольно странно придираться, что в книгах о трансформации фигур есть картинки, а в книгах по алгебре нет.
Опять же, Visual Group Theory содержит все те картинки, что должны были бы быть в учебниках по теории групп, но которых там почему-то нет. Потому, что картинки нельзя прочесть экранным диктором.
А в книгах по топологии так много картинок и диаграмм просто потому, что топологи полагаются на зрение (в самом деле, иначе бы они были бы алгебраистами), и потому хотят видеть.
>> No.160372  
Доброчан, я случайно нашёл годноту: https://www.cs.cmu.edu/~kmcrane/Projects/DDG/paper.pdf
>> No.160373  
>>160366
> У нас с тобой, похоже, разный Алюффи.
Ну, я доскроллил до ваших любимых "наглядных" картинок, а их я один хрен читать не умею. Припоминаю, для группоидов там что-то тоже было понарисовано, причем с абсолютно ебанистической кучей стрелочек.
>>160364
Вот я и хочу, чтобы вы меня убедили, что этот ваш язык полезен. Покажите, где он делает вещи проще* , а не сложнее. Когда я пытался этому научиться, у меня постоянно была мысль "нахуя всё это говно для описания того, что можно сказать в двух словах?!", на том и дропнул.
>>160365
Я слов-то таких не знаю.
>> No.160374  
>>160373
> нахуя всё это говно для описания того, что можно сказать в двух словах
Ага. Только вот если эти два слова расскрыть, получится суп из символов и таинственные заклинания вроде gX = Xg.
А ответ на главный вопрос жизни, вселенной и всего такого я и так знаю.
> я доскроллил до ваших любимых "наглядных" картинок, а их я один хрен читать не умею
> Я слов-то таких не знаю

Получается, ты даже самых-самых САМЫХ основ теорката не знаешь?
>> No.160375  
>>160374
> Получается, ты даже самых-самых основ теорката не знаешь?
Конечно! Я вообще довольно много всякой неведомой хуйни не знаю. Но в случае с теоркатом я просто уже всё забыл, ибо ни разу не пользовался.
И все же, убеди меня, что оно стоит того, чтобы его узнать. В результате решения каких задач возникает потребность в нём?
>> No.160376  
>>160375
> В результате решения каких задач возникает потребность в нём?
В принципе, я мог бы просто отправить тебя в https://ncatlab.org/ и https://en.wikipedia.org/wiki/Homological_algebra
Или сюда: https://arxiv.org/abs/1311.3903
Но скажу ещё пару слов: категории, как и множества — язык современной математики. Избегать их, наверное, можно, но с тем же успехом можно избегать английского языка. Но лучше основы всё-таки знать, благо они совсем просты.
>> No.160392  
>>160376
А доцент мехмата МГУ считает иначе.

> перечислить науки, для занятия которыми можно вообще не знать теории категорий.
Я хочу попробовать! Итак, начнем. Пойдем прямо по кафедрам.
1. Кафедра ТФФА. ТФДП-ники, школа которых берет начало от Лузина, Колмогорова, Меньшова, Бари если и используют категории, то только в смысле Бэра. :D ТФКП-шники одной переменной тоже их не используют. В большинстве разумных разделов ТФКП многих переменных, типа пространств функций в шаре, граничных свойств, геометрии областей и т.п категории не используются. В функане, теории операторов категории, если и используются, то сбоку-припеку, как новомодная дань, но новых результатов они не приносят.
2. Теория вероятностей и мат.статистика. Вот уж где нет категорий, так там.
3. ОДУ и УРЧП, Динамические системы. Категории не используются.
4. Общие проблемы управления. Категории не используются.
5. Дискретная математика и МАТИС. Категории не используются.
6. ОТГ, ВГТ, ДГи приложения - если категории и используют, то мал-мало, совсем не видно, нужно долго искать.
7. Матлогика и теория алгоритмов. Где же там категории??? Ау?
8. Кафедра теоретической информатики. Вот где категории живут? Или нет? :D
9. Теория чисел. Эти дня не проживут, чтобы категориями не воспользоваться! :D
10. Кафедра вычмата. Вот здесь без категорий точно все рухнет! :D
Фу, устал.

http://dxdy.ru/topic117119.html , страница 13.
>> No.160393  
>>160159
Есть ли хорошие калькуляторы с богатством функций wolframalpha, но в виде бесплатного софта для windows? Хотя бы просто хорошие калькуляторы.

А вопрос такой. Есть два длинных набора значений чисел. Какой формулой мне вычислить места, где наиболее близко стоят друг к другу число из 1-го с числом из 2-го набора?

Например, среди миллиона случайных чисел есть 1.834.295 из 1-го, и 1.834.201 из 2-го набора, и вот они-то мне нужны, а вручную искать долго.
В математике профан.

Реально ли сделать это с помощью экселя?
>> No.160394  
>>160392
> доцент
> мехмата МГУ

Ясно.
>> No.160395  
353_v1.png (0.0 KB, -1x-1)
0
>>160393
> Есть ли хорошие калькуляторы с богатством функций wolframalpha, но в виде бесплатного софта для windows?
Mathics, Python Notebook + куча библиотек.
> Реально ли сделать это с помощью экселя?
Пикрелейтед.
>> No.160396  
>>160395
А насчёт формулы не подскажешь?
>> No.160399  
>>160396
Нумеруешь, сортируешь и пробегаешь циклом. Ничего сложного.
>> No.160401  
>>160399
Я так не умею. Решил сделать проще, в таблице экселя значения первой группы пометил зелёным, а второй - красным. Совместил, отсортировал, и смотрел, где совпадают старшие разряды, затем младшие, и так, пока не на глаз не попадутся с наименьшей разницей.
>> No.160402  
>>160399
> сортируешь
Зачем?
>> No.160403  
>>160392
> А доцент мехмата МГУ считает иначе

Это не доцент, а хуент. Мне лень переходить по ссылке, но там небось какой-нибудь гнойный пидор с кафедры матанализа или какого-нибудь еще антинаучного говна. Отмечу только один пункт в его белиберде:

> 6. ОТГ, ВГТ, ДГи приложения - если категории и используют, то мал-мало, совсем не видно, нужно долго искать.

ВГТ - это кафедра, которой заведует филдсовский лауреат С.П.Новиков, чтобы вы понимали. На упомянутых кафедрах(кроме отг, где правда все грустно)процентов 80 людей используют аппарат теорката примерно в каждой статье. Это кафедры, где сидят Гусейн-Заде, Бухшатбер, Гайфулин и многие другие замечательные люди. И какой-то пидор с dxdy будет мне рассказывать, что там не знают теорката?
>> No.160404  
>>160392
Ну и ещё.

> В большинстве разумных разделов ТФКП многих переменных, типа пространств функций в шаре, граничных свойств, геометрии областей и т.п категории не используются.

Позволю себе напомнить товарищу доценту, что такая штука, как пучки, изначально были придуманы для решения задач комплексного анализа нескольких переменных(например,https://en.m.wikipedia.org/wiki/Cousin_problems). А работать с пучками без абелевых категорий - это... да невозможно, в общем. Или разве что переводить все утверждения с категорного языка на эквивалентный, но существенно более сложный. Так что я бы посмотрел на человека, занимающегося многими комплексными переменными без категорий.
>> No.160405  
>>160401
> Я так не умею
Но ты же математик! Найди решение.

>>160402
Это делает задачу значительно проще. Представь себе две параллельные прямые с выделенными точками. Двигая вдоль неё прямоугольное окно и попарно сравнивая те немногие попавшие в это окно точки, можно быстро найти ближайшие две.
Конечно, можно и втупую сравнивать, но это уже O(n^2) вместо O(n*log(n)).
>> No.160406  
screenshot.2017-06-30_23-39-17.png (0.0 KB, -1x-1)
0
>>160405
> Но ты же...
Вообще говоря, я ни разу не математик, и никогда не учился на математика, а в школе уроки прогуливал, поэтому, к сожалению, эти решения не для меня. Я увлекаюсь этими счётами просто так.
Но решение я нашёл, муахахаха! Пикрелейтед.
>> No.160408  
>>160392
https://lj.rossia.org/users/tiphareth/2071756.html
>> No.160409  
>>160408
> Категории были придуманы для того, чтобы внятно и коротко сформулировать (доказанную уже не тот момент) важную теорему: отображение Гуревича является естественным преобразованием. Для того, чтобы определить естественное преобразование, нужно определить функтор, чтобы определить функтор, нужно определить категорию. That's it. А вот вся эта хуйня, которую вы тут понаписали, это побоку и ни о чем.
Запомнил.
>> No.160423  
Короче, линкор с 9 стволами стреляет по вражескому кораблю, причём известно среднеквадратичное отклонение равное 2,1. Какой величиной является вероятность попадания снарядом по кораблю? Дискретной или непрерывной?
>> No.160424  
>>160423
непрерывной
>> No.160425  
-cSKGl8X9ts.jpg (0.0 KB, -1x-1)
0
>>160424
О, спасибо, но можно я проявлю наглость:
Линкор с 9 стволами имеет максимальный разброс на дистанции 23 км в 200 метров(внешний вид разброса круг), каковая будет вероятность попадания по вражескому кораблю, если среднеквадратичное отклонение будет равно 1.8, 1.9, 2.0, 2.1?
>> No.160522  
Платиновый вопрос, конечно, но подскажите хороший учебник/методичку по интегральному исчистлению, больше ориентированную на практику, т.е. задачами, примерами и всем таким, но также с определённым минимумом теории и обоснованием методов.
>> No.160524  
>>160522
Письменный сойдёт?
>> No.160526  
IMG_1095.jpg (0.0 KB, -1x-1)
0
>>160522
ня:
http://old.kpfu.ru/f6/k6/binfiles/integrm%2123.pdf
>> No.160529  
>>160526
>>160524
Пойдёт, спасибо большое!
>> No.160573  
На каких типах матриц тестировать алгоритмы, решающие системы линейных уравнений?
Как генерировать матрицы с большим числом обусловленности?
>> No.160584  
>>160573
Если говоря о типах, ты имеешь ввиду тип переменных, то мне на сиплюсплюс обычно хватало double.
Системы плохо обусловленны, если уравнения "почти" линейно зависимы. Можешь такие генерировать просто умножая одно из уравнений на постоянный коэффициент и делая небольшую отстройку.
2x+4y=3;
4.003x+7.992y=9.01.
Например
>> No.160585  
>>160584
Нет, типы матриц. Например, вот такие:
1 -1 -1 -1
0 1 -1 -1
0 0 1 -1
0 0 0 1
На них очень плохо работает всё (хотя вроде вовсе не линейно зависимы).
Спасибо за совет, попробую применить твой метод.
А как считать обусловленность матрицы? Можно найти обратную и посчитать её норму (и умножить на норму исходной), но это будет совсем неточно.
>> No.160586  
Кто мне пояснит такую вещь.

Сейчас в фаворе алгебра и алгебраическая геометрия (ну и теория чисел), я правильно понимаю? Что произошло с математикой ОТО (не знаю, как называется верно), теорвером, дискретной математикой? Чем вообще люди занимаются по сути?
>> No.160587  
>>160586
Есть международный сайт, на который математики выкладывают свои статьи. Если ты походишь по этому сайту, то увидишь ответы на свои вопросы и сформируешь правильную картину.
https://arxiv.org
>> No.160588  
>>160587
Не смешная шутка. На одну статью уходит от недели до месяца. И это я не учитываю работы, которые являются фундаментом отдельных теорий (600 далеко не простых страниц там обычное дело).
>> No.160589  
>>160588
В чем проблема читать абстракты?
>> No.160590  
>>160589
Глубины не чувствую.
>> No.160591  
>>160590
Твой вопрос - "чем вообще люди занимаются по сути". Посмотри на статьи, увидишь ответ. Другого ответа на такой вопрос не бывает.
>> No.160593  
>>160591
Бывает другие ответы. Например, вот таким занимаются http://stacks.math.columbia.edu/
У меня кругозор просто безумно узкий, вот и хочу спросить у людей других школ.
>> No.160595  
>>160585
Ты пример скинул вот матрицы. Я беру добавляю еще столбец справа с вектором свободных коэффициентов. Просто нули добавляю, получаю ответ, все переменные нули. Если свободный вектор немного отклоняю от нуля, порядка 0.01, то и ответ отклоняется примерно на 0.01 от нуля. Значит система хорошо обусловлена, все норм у меня работает с ней. Так что у нас что-то не сходится.
>> No.160602  
1500004103950.jpg (0.0 KB, -1x-1)
0
>>160587
Странное ощущение, похожее на наблюдение улья, в который пчёлы приносят мёд. И от мысли о том, сколько человекочасов надо потратить на учёбу, чтобы уметь читать эти статьи, становится как-то не по себе.
>> No.160611  
>>160602
> И от мысли о том, сколько человекочасов надо потратить на учёбу, чтобы уметь читать эти статьи, становится как-то не по себе.
Зависит от конкретной статьи, на самом деле.
>> No.160613  
V6LZTzmbeJMs2tzv_v0-vA.jpg (0.0 KB, -1x-1)
0
Умерла Мариам Мирзахани.
>> No.160614  
>>160613
Фигасе. Как так? Она же ненамного старше меня.
>> No.160615  
>>160614
Радуйся, что у тебя нет груди.
>> No.160616  
>>160615
У него всё ешё есть легкие и желудок.
>> No.160618  
>>160595
Если матрица 4 на 4, то всё хорошо. Но когда n увеличивается, то если, например, взять свободный вектор из одних единиц, то при уменьшении i x_i возрастает экспоненциально, и погрешность вроде бы должна расти так же.
В общем, возможно, я неправильно измерял точность, но у меня при увеличении n очень быстро растёт ошибка.
>> No.160623  
>>160618
То что ошибка растёт при увеличении это нормально. Но про экспоненциальность ты перегибаешь, не так все плохо. Я потестирую на своём коде твою матрицу сегодня и скажу свои результаты. Мне кажется все норм должно быть.
>> No.160625  
1-12.png (0.0 KB, -1x-1)
0
>> No.160627  
>>160625
5х5 = 25
(10n+5)(10n+5)=100nn + 100n +25
(100n+25)(100n+25)=10000nn + 5000n +625
и так далее. То же самое, начиная с 6 вместо 5.
>> No.160629  
>>160627
Я не понимаю как дать формальное доказательство.
>> No.160630  
>>160623
Вообщем, я нашёл свои пороги старые, чтоб по Гауссу считать. Бахаю n=20, сперва свободный вектор единички, потом 1.002. Решение изменяется на одинаковую относительную величину. На ноль ноль два в степени, степень меняется от 1 до n. Это вполне хорошо обусловленная система. Конечно можно вектор свободный подобрать такой, чтоб все стало плохо, но это не просто единички будут. Просто имей ввиду что обусловленность с ошибкой вычислений ничего общего не имеет. Если ты думаешь, что плохо обусловленную систему труднее решать, то ты ошибаешься. Просто смысла в решении меньше.
>> No.160631  
>>160630
Ах да, я понял, почему ты говорил про экспоненциальный рост. Там ведь сам ответ растёт как два в степени n, так что изменение тоже так растёт. Но ты имей ввиду, что относительное изменение в процентах остаётся одинаковым. Так что это нормально, не переживай.
>> No.160632  
>>160631
С одной стороны ты прав, но с другое стороны именно относительная погрешность растёт экспоненциально.
>>160630
А, всё, понял тебя. Ну хорошо, но мне такие вот матрицы тоже важны, которые может и хорошо обусловленны, но работают на них алгоритмы так себе.
Кстати, а как выбирать эпсилон?
>> No.160633  
>>160632
Эпсилон сам по себе не выберешь. Важна именно относительная погрешность. То есть отношение эпсилон к самому значению ответа выраженное в процентах. Если у тебя ошибка меньше 5%, то это приемлимо, если меньше 1% то это збс, если меньше 0.1% то это уже лучше чем зачастую надо. На основе того, какой у тебя ответ, ты можешь найти процент от ответа, это и будет эпсилон.

Я так делаю, потому что на физике приучили. Возможно брать эпсилон с потолка, так сказать по точнее, с запасом и все.
>> No.160891  
А правильно ли я понимаю, что метод Якоби и метод итерации -- одно и то же?
>> No.160904  
Z7zEy3BfDnQ.jpg (0.0 KB, -1x-1)
0
Привет доброаноны, вот попалось такое задание. Мои мысли: необходимо рассмотреть случаи, раскрывая модули с разными знаками, в итоге получить много промежутков, после чего их просто объединить. По-моему данный способ слишком нерациональный, может кто знает как это нормально решить?
>> No.160905  
>>160904
> промежутки
Значения, пофиксил себя.
>> No.160909  
>>160904
Для каждого промежутка ты тут же вычисляешь минимальное значение на этом промежутке.
А так да, рассматриваешь 8 вариантов раскрытия, получаешь сколько-то чисел (меньше 8) и бере минимальное.
>> No.160911  
>>160904
>>160909
Этому треду не хватает олимпиадников.

Рассмотрим оцениваемую величину как функцию двух переменных, х и у. График этой функции, очевидно, состоит из нескольких(на самом деле семи) плоскостей с ребрами в местах, где один или два модуля зануляются. По геометрическим соображениям очевидно, что минимум такая функция может принимать либо на ребре, либо на грани графика, параллельной плоскости Оху. У данного графика такой грани нет, как легко понять. Остаются ребра. Каждое из ребер - прямая, на которой исследуемая величина превращается в функцию одной переменной, которую можно оценить непосредственно.
>> No.160912  
>>160904
"Чего тут думать -- прыгать надо!"

Прыгать не надо.

1. Проведи мысленно три прямые на плоскости вида <выражение под модулем> = 0.
2. Возьми точку вне этих прямых. В окрестности этой точки твоя функция является неконстантной линейной функцией (почему?), а стало быть не достигает минимума (почему?).
3. Теперь рассмотри точку, принадлежащую ровно одной из трех прямых. Пропатчи рассуждение выше для этого случая.
4. ...
5. PROFIT!
>> No.160915  
pr8HyrtsDts.jpg (0.0 KB, -1x-1)
0
Спасибо за ответы, добра вам.
>>160911
Я закончил 9-ый класс и еще не совсем представляю, что такое ребра и грани графика , извини, а это задание просто в ЗФТШ в конце попалось, а так как я очень любопытный, конечно же под звездой все решаю, ну или пытаюсь по крайней мере. Если бы я понимал вышесказанное тобой, то наверное тебя бы зацеловал за столь умный ответ, спасибо.
>> No.160918  
>>160915
Ты, наверняка, знаешь, что такое график функции одной переменной. У функции двух переменных тоже есть график - это поверхность в трехмерном пространстве. Если мы имеем дело с функцией одной переменной, то график суммы нескольких модулей функций вида ах + b представляет из себя ломанную с углами в местах, где какой-нибудь из модулей зануляется(например, у |x| один излом в нуле, у |x-1| + |х| - два угла в нуле и единице, и т.д.). Если же перейти к двум переменным, то график суммы модулей функций вида ax+by+c - это многогранник(правда, некоторые грани у него бесконечные), у которого ребра в местах, где зануляется один из модулей, а вершины - там, где зануляются два или больше модулей(впрочем, зануления трех или больше модулей не бывает, если функция хорошая). В случае функции одной переменной минимум сумма модулей линейных функций(а именно так называются функции вида ax+b и ax+by+c) может принимать только в углах графика(это понятно из геометрических соображений). То же самое, если подумать, верно и для двух переменных - самая низкая точка на многограннике должна быть в углу, на вершине. Поэтому тебе нужно рассмотреть только вершины графика, а это те точки, где зануляются по два модуля.
>> No.160919  
>>160918
Ого, не думал, что мне будут так хорошо объяснять на анонимном форуме такие умняши, как ты. Я удивлен, но я понял твое объяснение и ты, черт возьми, прав. Вот теперь я точно готов тебя расцеловать, еще раз большое спасибо.
>> No.161100  
15002410052680.jpg (0.0 KB, -1x-1)
0
Не могу решить простенький дифур, вот тут описал вопросы
https://pastebin.com/THrBrZP3
>> No.161101  
>>161100
Ты там что-то сильно мудреное пишешь, а все просто.

Прочитай в учебнике, как решаются неоднородные линейные диффуры(каковым твой является). Решение является суммой решения однородного уравнения и какого-нибудь частного пешения неоднородного. Ты как раз нашел частное.
У однородного уравнения решений целое линейное пространство размерности два, за исключением тех случаев, когда уравнение вырождается(т.е. у характеристического полинома кратные корни). Этот-то случай тебя и интересует, тогда, как легко понять, получается единственное решение.
>> No.161105  
>>161101
> Решение является суммой решения однородного уравнения и какого-нибудь частного пешения неоднородного.
Спасибо, теперь то все сошлось, за лето забыл про частное- общее, помню меня еще напрягло слово частное, но я мимо прошел.
А поп поводу задачи Коши я прав? Можно как-то применить тут теорему о единственности при непрерывности?
>> No.161106  
>>161105
Это не задача коши, у тебя недостаточно начальных условий.
>> No.161109  
>>161106
Да, я про это написал. Но я так понимаю, что теоремы о существовании и единственности решения задачи Коши n порядка не существует. Я прав?
>> No.161111  
>>161109
Ты вообще все забыл. Задача коши n- того порядка заменой переменных сводится к задаче коши первого порядка. Поэтому теорема существования и единственности там такая же. Но необходимы начальные условия для всех переменных, т.е.(если заменить переменные обратно)для всех производных до n-1 порядка. У тебя есть начальное условие только на саму функцию, а для теоремы единственности нужно еще и на производную.
>> No.161112  
>>161111
Начет замены переменных не знал. Т.е. просто вводим н-1 новую переменную и получаем задачу коши для вектор-функции, для которой решение существует и единственно при некотором условии типа непрерывности.Про начальные условия я помню, наступал уже на эти грабли. Но я к тому, что раз у нас есть только одно начальное условие, то второе мы можем выбрать сами и для любого такого будет решение => мы можем получить сколько угодно решений. Но это же противоречит

> Этот-то случай тебя и интересует, тогда, как легко понять, получается единственное решение.
>> No.161113  
>>161112
В вырожденной точке у тебя уравнение не зависит от y. Поэтому заменой переменной z = y' оно превражается в уравнение первого порядка. Поэтому нарушения теоремы единственности нет, хотя кажется, что оно есть: в особой точке нам достаточно одного начального условия.
>> No.161114  
>>161113
Хм, действительно, теперь все стало понятно, спасибо.
>> No.161138  
>>161100
>>161101
>>161113
Чушь пишете-с. У линейного уравнения решения всегда образуют аффинное пространство размерности, равной порядку.

Решать надо так:
1. К этому уравнению применима теорема существования и единственности.
Если это не очевидно или ты знаешь эту теорему только для систем первого порядка, то
>>161111
вот этот товарищ дело говорит: переходим к системе уравнений первого порядка и убеждаемся, что условие теоремы выполнено.
2. Из вышеупомянутой теоремы следует, что для любого p начальное условие y(2) = 6, y'(2) = p дает единственное решение. Т.е. решений столько же, сколько p, т.е. однопараметрическое семейство.

Стало быть, ответ -- ни при каких.

Заметь, кстати говоря, что при a=0 ты можешь получить общее решение, просто дважды интегрируя правую часть. При этом вылезают две константы, а у тебя одно условие y(2) = 6, которым ты обе константы не убьешь. Так что это случай, когда без всякого знания теории как раз очевидно, что решение твоей задачи не единственно. (Хотя я это больше говорю для товарища >>161101, который писал про вырождение и легко понять , лол).
>> No.161140  
У фрактала того самого знаменитого, похожего на грушу бесконечный периметр? А площадь? То есть, это же бесконечная сумма каких-то все более мелких штук. Там не получается как с бесконечно мелкой геометрической прогрессией?
>> No.161142  
>>161140
Площадь конечна, периметр бесконечен.
>> No.161145  
>>161138
Ну вот это ложится нормально на мое представление. А то я бьюсь уже который день, силясь понять, почему такая бяка при а=0 и решение единственно.
>> No.161151  
Анон, новый вопрос из той же оперы.
y''+ (a^2)y = tg(x)
При каких а существует единственное непродолжаемое решение с областью определения (-1;3/2).
Я так понимаю, что раз все функции на этом промежутке непрерывны => мы по старым схемам можем поставить сколько угодно задач Коши и каждая будет решаться. И опять ответ ни при каких.
>> No.161171  
>>161151
В общем я подумал посидел, не нашел ошибок в рассуждении. Меня смущает, что все так просто.
>> No.161295  
Мне на матане, когда говорили про полный дифференциал функции двух переменных, говорили что его можно записать нормально, а можно через символический оператор* (6 — круглое d): df = (6/6x dx + 6/6y dy)f.
Это породило у меня несколько вопросов.
Как правильно называется этот оператор? А то в гугле только наблы. Или "символический оператор" == "оператор"?
Круглая d — это строчная дельта?
Что такое круглое d, и чем оно отличается от нормального? Нам сказали, что нормальное используется, когда речь идет о полной производной, а круглое — для частных. Почему так?
>> No.161296  
2017-09-09-193248_3200x1080_scrot.png (0.0 KB, -1x-1)
0
Кстати, ошибался ли у вас вольфрам? У меня, вот, позавчера он сказал, что придела нет, а он есть и равен нулю (Демидович со мной согласен, правда в этом переиздании огромное количество опечаток, в том числе и в ответах). Вроде, забил в вольфрам все правильно. Или я что-то не понимаю?
>> No.161306  
>>161295
dabba
>> No.161312  
>>161296
Вольфрам все отвечает правильно. Дело в том, что если считать х и у комплексными, то предела действительно нет - если идти в бесконечность по вещественным х, у, то получается ноль, а если брать х и у чисто мнимыми, то е^(-х-у) при стремлении к бесконечности остается ограниченным, а х^2+у^2 возрастает, т.е. предел - бесконечность.
>> No.161313  
>>161312
Ясно.
>> No.161320  
Screenshot_3.png (0.0 KB, -1x-1)
0
>>160159
Добра вам. Можете помочь с двумя не очень сложными проблемами.
1. Попытался обозначить за t, в итоге выходит выражение с корнями, попробовал возводить в квадрат, получились запредельные числа, пробовал даже брать оба корня в знаменателе за A и B, но все равно не получалось. В общем мне нужно каким-то образом найти этот промежуток t>=16, а потом просто подставить его и получится корень x = 7 (комплексный).
2. Я сделал для двух случаев, когда (b < 0) => корней нет, (b = 0) => точка (0;0). Последний случай, когда (b > 0) , вот там я и затрудняюсь.
>> No.161324  
>>161320
1) выражения в знаменателе - квадратные корни из -(х-7)^2 + 16 и
- (х-7)^2+4, соответственно. Поэтому левая часть неравенства достигает минимума при х=7, и, как легко проверить, этот минимум и равен 13/2. Поэтому неравенство выполняется всюду, где определены дроби.
2)При b>0 график функции х= -|b-y^2| - модуль параболы, т.е. он выглядит, грубо говоря, как буква м, положенная на правый бок. График у = а(х+b^2) - прямая. Если ее точка пересечения с осью 0у оказывается внутри отрезка, отсекаемого на оси графиком х = -|b-y^2|, то корни обязаны быть, так как область внутри графика и слева от оси ограничена, и прямая должна из нее выйти. Если точка пересечения выше, то это значит, что а>0, т.е. тангенс угла наклона прямой положительный. Поскольку тангенс угла наклона верхней ветви графика х= -|b-y^2| отрицательный, то корень быть обязан. Аналогично дело обстоит в случае, когда точка пересечения оказывается ниже отрезка. Поэтому подходит любое b>0.

Я думаю так.
>> No.161325  
>>161324
Перепутал в первом со знаком неравенства, но идея от этого не меняется.
>> No.161330  
>>161324
Я даже не знаю какими словами описать, как ты мне помог. Спасибо большое, хочется тебе вручить денежный купон на миллион долларов, жаль у меня нет таких денег. :(
Могу пожелать лишь добра, надеюсь все твои желания сбудутся, а я смогу мыслить и решать так же. Еще раз спасибо.
>> No.161332  
>>161330
Ты посредством меня олимпиаду решил, что ли?

Не за что, мне не жалко.
>> No.161334  
>>161332
Нет, олимпиады еще не стартовали. Это 2 задания , которые я не смог решить в школе "N".
>> No.161335  
Аноны, че то вот да, вопрос есть. Проходим двойные интегралы и там тема про "площадь плоской хуйни=двойной интеграл по dxdy". С одной стороны это классно, да, единица это нихуя, и по определению интеграла мы просто получаем сумму маленьких площадей, что дает нам площадь. Но с другой стороны это дает нам объем цилиндра с площадью основания, которая нам нужна, и высотой 2. Это немного напрягает мой мозг. И в физических приложениях разных возьмем, например, единичный куб. Его длина 1 метр, площадь 1 метр квадратный, объем 1 метр кубический. И это имеет смысл. Длина это метр, площадь это метр помножить на метр и объем, ну понятно. Это можно представить. Но если сделать это в 'голой' математике, которая идет у нас на парах, то у нашего куба длина будет 1, площадь 1, объем 1. Без единиц измерения это выглядит как минимум странно. Кто-нибудь может сделать что-нибудь, чтобы это не напрягало мой мозг?
>> No.161336  
>>161335
Ещё, в первом курсе нам выдали 'Основы математического анализа' Позняка. А в этом нет, но я скачал обе части. Почему в них нет ничего про дифференциальные уравнения? Почему вообще линал и анал. геометрия не часть математического анализа?
>> No.161337  
>>161335
Пойми, что любые единицы измерения - чистая условность. Можно считать в метрах и литрах, а можно в ярдах, галлонах и кубических футах. При этом математические формулы остаются верными в любых системах измерений. В этом и состоит их ценность.
>> No.161343  
>>161336
Наверное потому что это другие разделы математики? Почему мотоцикл - это не один из автомобилей?
>> No.161355  
Пытаюсь разобраться в торических многообразиях, но идёт туго. То есть я могу читать и понимать определения, но сильно легче не становится. Бэкграунд у меня дифференциально-геометрический. Может кто-нибудь вкратце пояснить? То есть меня интересует, как с ними работать, производить вычисления (наверное в однородном координатном кольце) по веерам и политопам и такие штуки. Хотелось бы, конечно, что-то в духе обычного описания проективных пространств.
>> No.161357  
>>161355
Тебе зачем? В торические многообразия можно заходить с разных сторон.
>> No.161360  
>>161357
Хочется писать там какие-нибудь явные уравнения и производить явные вычисления (inb4 зачем), как в проктивных. Как я написал, хочется зайти с диф-гем стороны)
>> No.161363  
>>161360
> inb4 зачем

Нет, а все-таки? Есть ощущение, что если у тебя возникают такие позывы, то ты не совсем понимаешь, что такое торические многообразия и зачем они нужны. Потом, а в чем сложность? Торическое многообразие покрывается картами, берешь локальные координаты и пишешь, что хочешь. Замены координат на торических многообразиях простые, ненамного сложнее, чем в проективных пространствах.

Энивей, вот один хороший источник по торическим многообразиям:

http://m.mathnet.ru/php/archive.phtml?wshow=paper&jrnid=rm&paperid=3391&option_lang=rus

Рекомендую также посмотреть статьи, цитируемые Даниловым, равно как и статьи, цитирующие Данилова. Можешь увидеть много примеров того, что можно назвать "явными вычислениями" в торических многообразиях.
>> No.161366  
>>161355
Можешь пояснить, как начать вкатываться в дифференциальную геометрию? У меня верх матана - многократные интегралы, интегралы второго рода (векторные которые), всякие интегральные преобразования. В общем, стандартный курс у физиков. Куда дальше идти?
>> No.161367  
>>161366
Второй том Зорича для самых основ, затем “Геометрические методы” Шутца и “Математические методы” Арнольда.

другой анон
>> No.161369  
13715329_630110520486326_607760048_n.jpg (0.0 KB, -1x-1)
0
>>161367
Спасибо, хорошо, что счас выходные епта. Буду наворачивать.
>> No.161371  
ZrHgbsEKTHg.jpg (0.0 KB, -1x-1)
0
>>161363
Спасибо, посмотрю. Ну тут скорее меня интересуют не столько сами торические многообразия, а они как арена для других, всякие поверхности в них. Как-то очень непривычно пока описывать всякие объекты в терминах комбинаторики вееров и многогранников. Как будто совершенно другой язык без всякого перехода.

>>161366
Эх, давно это было. Во-первых, особенно если ты физик, надо отбросить кучу не особо нужных фактов из анализа (в духе кучи разных определений разных интегралов, вопросы сходимости и гладкости и прочее). Во-вторых тут, на мой взгляд, надо понять несколько концептов (упорядочил так, как бы я их изучал):

1) Многообразия и работа с картами. Все правильные понятия анализа по-настоящему имеют геометрическую формулировку, и базовых понятий очень мало на самом-то деле. Любой анализ можно устраивать на любом кривом пространстве, работая в окрестностях точек в координатах, а затем склеивая.

2) Интеграл. Все интегралы (одномерный/многомерный/криволинейный/...) с учётом пункта выше сводятся к одному пнятию - дифференциальная форма, это очень важный концепт, особенно если ты физик. Очень рекомендую по ним главу "Дифференциальные формы" в мат. методах класической механики Арнольда.

3) Связано с предыдущим - многие понятия дифференцирования (производная, градиент, ротор, дивергенция, ...) и неопределённого интеграла являются одним концептом - внешняя производная дифференциальной формы. Все формулы типа Грина/Гаусса-Остроградского/Стокса сводятся к очень простой общей формуле Стокса, классно написано в Арнольде. Рекомендую посмотреть электродинамику, записанную через дифференциальные формы.

4) Как физику тебе также очень понадобятся понятия расслоений (калибровочных теорий), связностей (калибровочных полей) и ковариантных производных, а также групп Ли (симметрии в физике), но в это можно вкатываться постепенно. У Болибруха была классная видеолекция на тему того как он переформулировал классический электромагнетизм начиная с уравнений Максвелла, потом через дифференциальные формы, а затем и через связности. Очень рекомендую, в любом случае изучать можно с точки зрения ЭМ.

Дальше можно двигаться в сторону большей геометризации, изучать глобальные аспекты, когомологии (токи без источников), харклассы (топологические заряды) и кучу всего другого, но это приложится, если захочется.

Как книги я бы не рекомендовал Зорича. Несмотря на то, что он довольно хорош, он весьма затянут, и в нём сделан упор не на те вещи, на мой взгляд.
Я бы порекомендовал опять же отдельные главы в матметодах Арнольда; как справочник "Современная (уже нет:D) геометрия" Дубровина, Новикова и Фоменко. Написана специально, как объединяющий мост между физиками и математиками, с ней идёт более новая книга Новикова и Тайманова. Последняя больше подойдёт в качестве учебника. Ну и в интернете есть куча записок всяких лекций.

тот анон
>> No.161379  
>>161371
> как справочник "Современная (уже нет:D) геометрия" Дубровина, Новикова и Фоменко
Лучше сразу Спивака, он и полнее, и читать его приятнее.
>> No.161380  
>>161371
> Как-то очень непривычно пока описывать всякие объекты в терминах комбинаторики вееров и многогранников. Как будто совершенно другой язык без всякого перехода.

Ну так на то это и алгебраическая геометрия, чтобы все описывать в алгебраических терминах. Ты геометрией над конечными полями не интересовался? Прикинь, у кривой над конечным полем (т.е. у конечного множества точек)есть кривизна и кручение, причем из их исследования многое получается. А также они бывают гладкими и негладкими. Конечно, все эти понятия определяются совершенно негеометрически.

По поводу дифгема - от себя посоветую "алгебраическую геометрию" Гриффитса-Харриса. Хоть это и книга по алгему, но он там рассказан максимально геометрично и с привлечением всего аппарата дифгема.
>> No.161384  
1497463952608.png (0.0 KB, -1x-1)
0
>>161371
Спасибо большое. Начал смотреть Зорича, пока вроде по силам, но я чувствую, что без иллюстраций (хотя бы лекций на ютубе) будет долго в памяти укладываться. Здорово увеличил мою мотивацию с этими дифференциальными формами, хех. Алсо, насчет групп Ли, ты так внезапно берешь и тут их упоминаешь. Их действительно стоит изучать уже после всего, что ты тут перечислил или это не важно? Я конечные группы частично освоил, так у меня в учебнике (Любарский, теория групп и ее приложение к физике) сразу хуячат группы Ли и непрерывные группы (их не трогал еще). Ну с намеком, что нихуя кроме конечных групп знать не надо, чтоб группам Ли переходить.
>> No.161388  
>>161384
> Ну с намеком, что нихуя кроме конечных групп знать не надо, чтоб группам Ли переходить.
Ну основы (топология, многообразия, координатный базис, касательное расслоение) то нужны.
Вообще, читай Шутца. У него всё очень ясно написано, да ещё и с картинками.

Кстати, если любишь программирование, есть вот такая лютая годнота:
https://www.cs.cmu.edu/~kmcrane/Projects/DDG/
https://www.cs.cmu.edu/~kmcrane/index.html
>> No.161389  
1498098661286.jpg (0.0 KB, -1x-1)
0
>>161388
Литературы уже добрый список собрал хех. На месяц вперед хватит. Спасибо еще раз, DISCRETE DIFFERENTIAL GEOMETRY наверну тоже, но думаю после того как в терминологию войду получше. Может между делом с вопросами буду сюда забегать.

Вот, например. На пикрилейтед, это из Спивака, говорится что лямбда - это теперь будет производная. Как она с обычной производной соотносится? Или это D f - отдельная от f' вещь и они никак не соотносятся?
>> No.161390  
>>161389
> Как она с обычной производной соотносится?
Положи n, m = 1 и увидишь.
>> No.161392  
>>161390
Тааак, понял. А то что лямбда линейное отображение, это означает, что лямбда всегда имеет вид A*h+B?
>> No.161393  
>>161392
Это означает, что λ — линейное отображение. https://ru.wikipedia.org/wiki/Линейное_отображение
Суть ведь проста: мы хотим знать поведение f в окрестности точки. Оно, вообще говоря, сложно. Но если f дифференциируема, а окрестность мала, то f(a + h) примерно равно f(a) + λ(h) — линейное отображение.
Линейные отображения просты и понятны, за что их все любят.
>> No.161419  
задача.png (0.0 KB, -1x-1)
0
Красота математики... Увы, но иногда мне приходится возвращаться в физику. А в физике приняты вещи вроде пикрелейта.
Вроде бы элементарное задание... Однако, я до сих пор не могу понять что бы интеграл от (n крест grad φ) dS мог бы означать. Вроде бы каждый символ в отдельности и понятен, но при этом в выражении в целом как будто бы нет никакого смысла.
Я хочу овладеть диффгеомом и забыть ЭТО как страшный сон, но в данный момент я вынужден* решать подобные задачки, а стало быть, мне придётся разуметь несколько слов на оркском.
Однако, мне нужна помощь.
>> No.161421  
>>161419
n - это единичный вектор нормаль для поверхности по которой интегрируешь, его иногда еще заносят в dS. Получают dS - вектор длинной с элементарную площадку в направлении нормали.
grad - тоже вектор.
x - векторное произведение n на grad. В результате тоже вектор, который интегрируется по dS.
Но я думаю ты все это и так сам понимаешь. Тогда в чем вопрос?
https://ru.wikipedia.org/wiki/ФормулаГаусса—_Остроградского
Вот эта формула используется, только в другую сторону.
>> No.161427  
>>161421
> x - векторное произведение n на grad. В результате тоже вектор, который интегрируется по dS.
И результат подобного интегрирования — тоже вектор?
>> No.161428  
1496872744716.jpg (0.0 KB, -1x-1)
0
>>161427
Бля, теперь тоже не понимаю что там написано. Может там покомпонентно подразумевается? Либо dS тоже вектор (должно быть жирным выделено) и там скалярное произведение. Ты смотрел контекст? Там перед заданием может какие-то указания к подобному дерьму. Я тоже эту хуету на парах никогда не понимал как следует. Ну то есть когда мне математик рассказывал про формулы Стокса, Остроградского и так далее все было норм, я довольно ловко брал всякие потоки через поверхности натянутые на гиперболоиды. Либо скалярную величину в итоге интегрируешь, либо интеграл второго рода. А вот потом, когда один физик что-то мне пояснял подобное, всякие формулы для комбинации оператора набла и все вот это. Честно говоря я нихуя так и не врубился, там кажется было некое Правило Лагранжа или еще какая-то эзотерика. Это как раз тот случай, когда физики обсираются тут на мой взгляд. Я так понял, ты хейтишь все эти неформальности у физиков, надеюсь ты не имеешь какого-то предубеждения насчет них. Я просто тоже физик лол.
>> No.161429  
>>161428
Апдейт
Походу там поток в твоем примере все таки считается.
https://ru.wikipedia.org/wiki/Потоквекторногополя
Раз там нормаль вынесена отдельно, то dS все таки не вектор. Тогда X должно быть не векторным, а скалярным. Хз, может тут такие обозначения ебанутые, такое возможно в принципе. Хотя выше же через точку скалярное обозначается... Но вообще, поток больше всего подходит.
>> No.161430  
>>161428
Батыгин, Топтыгин «Сборник задач по электродинамике»
> Там перед заданием может какие-то указания к подобному дерьму.
Однако же нет.
>> No.161431  
>>161428
Судя по всему, там всё-таки должен получаться вектор при интегрировании. Осталось только выяснить, как это выразить на языке дифф. форм.
>> No.161452  
>>161419
Анон, я рандом, которого можно и не слушать. Но вот я подумал, и что придумал, не уверен, что это ответ на твой вопрос, но всё таки. Поправь, если где-то ошибаюсь. Это два интеграла, которые вычисляют поверхность. По замкнутому контору: берётся единичный вектор dl , он идёт по контуру и складывается, чтобы им замостить поверхность, мы берём φ, получаем дофига контуров, один большой и внутри много маленьких. Их все суммируем и наша поверхность замощается. Во втором случае использует n , который перпендикулярен площадки и по модулю равен площади площадке. Берут очень много малых площадок внутри контура, и складывают, но нужно знать, где границе контура, для этого используют градиент. То есть я представляю так, у нас очень много точек, и градиент это вектор по всем направлениям от центра, таких векторов очень много и начинаем замощать всё вокруг, но когда мы доходим до границе, то есть контура, мы за неё перейти не можем, так как градиент нам это не даёт (там он 0) Дальше мы просуммируем все точки и получаем поверхность, которая и ограничена контуром.
>> No.161466  
1-41.jpg (0.0 KB, -1x-1)
0
Анон, есть одна контрольная. Поясни, как сделать 1 номер, а именно, как сделать обратный порядок интегрирования в полярных координатах. Я понимаю, что суть двойного интеграла в том, что мы как бы фиксируем 1 переменную, и смотрим как изменяется другая. Зафиксировали x, y будет от y2 до y1, зафиксировали y, х будет от х1 до х2, зафиксировали fi, r будет от r1 до r2. А в обратном порядке в ПСК? Мы фиксируем r и меняя угол вырезаем такие вот кусочки? Я правильно сделал весь номер?
>> No.161479  
>>161466
Bump!!!
>> No.161504  
Доброчан, есть одна комбинаторная задача, которую я не понимаю как решить.
Суть: есть строки длиной n составленные из k символов (с повторениями). Сколько строк содержат все k символов?
>> No.161505  
>>161504
Всего строк n^k. Строк, не содержащих первый из k символов (k-1)^n, и столько же для второго, третьего,..,k-того символа. Однако, если мы просто вычтем из k^n k (k-1)^n, то мы дважды выкинем, например, строку, не содержащую первого и второго символа - когда выкидывали строки, не содержащие первый символ, и когда выкидывали второй. Поэтому нужно еще прибавить (k(k-1)/2) (k-2)^n. Но тогда окажется, что мы трижды посчитали все строки, не содержащие каких-нибудь трех элементов, т.е. нужно еще вычесть (n(n-1)(n-2)/6)*(k-3)^n строк. Ну и т.д.
>> No.161506  
>>161505
Прибавить, отнять, затем прибавить, затем отнять... Я надеялся, что есть простой способ, позволяющиё избежать кошмара с формулами.
>> No.161507  
>>161506
Мне он неизвестен, по крайней мере. Не думаю, что есть другой, ибо задача довольно искуственная, и вряд ли кто-то особо над ней задумывался, чтобы считать другими способами.
>> No.161511  
Как сдать теорию по дифурам?
>> No.161512  
>>161511
Можно попробовать ее выучить, например.
>> No.161528  
Какие есть парадоксы логик/теорий множеств, из тех, что можно объяснить на пальцах, кроме парадокса Рассела, Нельсона, лжеца?

Алсо, в соседнем треде очень не любят аксиому выбора. Мне интересно, что в ней плохого? А то я совсем не в теме.
>> No.161533  
>>161528
Про парадоксы, которые можно рассказать на пальцАх, я знаю мало что. А вот про аксиому выбора...

С одной стороны, аксиома выбора(или эквивалентные утверждения, вроде леммы Цорна)позволяет легко и просто доказать некоторые крайне нужные утверждения, вроде теоремы Хана-Банаха в функциональном анализе или, например, теоремы о том, что пересечение всех идеалов коммутативного кольца - нильрадикал. С другой стороны, она имеет неприятные следствия - например, парадокс Банаха-Тарского или то, что существуют подмножества вещественного линейного пространства, не имеющие объема. С третьей стороны, математика без аксиомы выбора выглядит непривычно и неуютно - так, в обычной математике вещественные числа упорядочены(т.е. то, что мы можем говорить, что, например 2>1) есть именно следствие аксиомы выбора - без нее порядок не всегда можно построить. Или, например, такой пример: аксиома выбора эквивалентна утверждению, что декартово произведение непустых множеств непусто. С четвертой стороны, аксиома выбора логически независима от остальных аксиом Цермело-Френкеля, а потому просто выбросить ее недостаточно - тут "всю систему менять надо". Так что вопрос это интересный и неоднозначный.

Однако, с пятой стороны, не стоит думать, что вопрос этот так уж важен. Рекомендую почитать дискуссию по ссылке:

https://sowa.livejournal.com/92839.html#comments

> Большинство математиков не только не работает в ZFC, но и не знает, что это такое. Нет никаких оснований думать, что ZFC исчерпывает нашу интуицию.

> Мне трудно представить, где бы можно было найти пример формального доказательства, более сложного, чем самые первые следствия из аксиом теории множеств или арифметики Пеано.

> Я уже несколько раз (тут) писал о том, что математики даже не знают аксиом ZFC. Их этому не учат. Вот вы знаете все эти аксиомы? В частности, ту, в честь которой стоит буква F (интернетом и книгами не пользоваться!)?

Если завтра люди, занимающиеся основаниями математики выпустят положуху, где будет сказано, что аксиомы ZFC - зашквар, а нужно пользоваться другими, истинными аксиомами, математический мир, вероятнее всего, не обратит на это никакого внимания.
>> No.161534  
Анон, а какие твои любимые функции? Мне вот очень понравилась f(x)=x*10^(-[lg(x)]-1), которая делает из x 0.x
>> No.161535  
>>161533
Для современной математики NBG подходит лучше, чем ZFC, ибо теоркат.
>> No.161536  
>>161535
Опять-таки, that's not the point. Никто этих аксиоматических систем не знает, и никто не доказывает теоремы, сводя их к аксиомам той или иной системы. "Подходит" в каком смысле?
>> No.161537  
>>161536
Это не есть что-то плохое, это называется "специализация". Каждый занимается чем-то своим. Основаниями занимаются теоретики оснований - вряд ли они смогут так же легко посчитать когомологии какой-нибудь загогулины, как специалисты по когомологиям, но это не делает когомологии чем-то ненужным в математике. Симметрично с основаниями - если теоретик когомологий не может сходу перечислить все аксиомы NBG, это не значит, что NBG абсолютно не нужна. Каждому - свое дело и своё занятие, вот и всё. К тому же все математики были студентами, и все изучали стандартные наборы аксиом. При необходимости любой работающий математик может восстановить в памяти ZFC или NBG самое большее за полчаса.
>> No.161547  
>>161537
Я и не говорю, что это плохо.

> При необходимости любой работающий математик может восстановить в памяти ZFC или NBG самое большее за полчаса

Интересно, с чего бы у него возникла такая необходимость.
>> No.161548  
>>161537
> К тому же все математики были студентами, и все изучали стандартные наборы аксиом

Тоже очень спорное утверждение. Я с ходу не могу назвать ни одного базового учебника теории множеств(Бурбаки не предлагать, это не учебник, да и они тоже этого не делают, в общем-то), в котором бы что-то выводилось из аксиом ZFC(равно как и никто не выводит геометрию из "честных" аксиом Гильберта). Аксиомы Пеано бывает, используют, но это тоже экзотика. Если они где и встречаются, то в курсе мат. логики - т.е., как уже говорилось, знакомятся с ними только те, кто специально интересуется основаниями математики.
>> No.161552  
>>161548
> Я с ходу не могу назвать ни одного базового учебника теории множеств
«Не совсем наивная теория множеств»?
>> No.161554  
>>161548
> Я с ходу не могу назвать ни одного базового учебника теории множеств
Проблемы в твоем кругозоре. Jech, Set theory third millenium edition. Мендельсон, математическая логика. Зорич, "Математический анализ".
>> No.161575  
1352244258445.png (0.0 KB, -1x-1)
0
>>161380
Ну это да. Так-то интересовался, как же, это на мой взгляд очень красиво, что можно работать с геометрией в конечных полях, и наоборот (гипотезы Вейля там всякие). Но на мой взгляд у всех этих геометрических терминов хоть и кардинально меняются определения, смысл остаётся такой же. И я продолжу с ними работать как и с понятиями из аналитического мира, хоть и со своей спецификой и дополнительными возможностями. А в торической науке потихоньку разбираюсь, действительно интересно, но опять же, для этого приходится делать некоторый "словарь".

Я бы сказал, что Гриффитс-Харрис - это именно книга по алгебраической геометрии для дифференциальных. А человеку нужна, видимо, по дифференциальной для физиков!

PS теорию Черна-Вейля там ведь можно построить? (если правильно понимать интегралы и не брать в расчёт всякие патологии).
>> No.161576  
0001.jpg (0.0 KB, -1x-1)
0
>>161419
ДНФ и Арнольд помогут переводить с орочьего) Сама постановка задачи странновата, но её тоже можно решать на языке дифф. форм (как в приложенном файле). Там я расписал очень подробно формулы (3), (4) после пары раз считаются в уме.
>> No.161577  
>>161528
Фраза "натуральное число, которое нельзя определить фразой русского языка, содержащей менее чем миллион букв", парадоксальна.
>> No.161578  
>>161575
> PS теорию Черна-Вейля там ведь можно построить? (если правильно понимать интегралы и не брать в расчёт всякие патологии)

Я о таком ничего не слышал и, признаться, не понимаю зачем бы она была нужна. К построению такой теории есть серьезное препятствие: в современной алгебраической геометрии многообразие понимается не как конкретное множество, а как класс эквивалентности относительно изоморфизмов, или даже более абстрактно, как некий носитель для пучков, сам по себе эфемерный. А как по такому интегрировать?

> Но на мой взгляд у всех этих геометрических терминов хоть и кардинально меняются определения, смысл остаётся такой же.

Мне, честно говоря, увидеть такой единый смысл не всегда удается. Например, понятие нормальности многообразия: геометрически оно означает, что подмногообразие особенностей имеет коразмерность не менее двух(т.е. нормальная кривая - это гладкая кривая, а нормальная поверхность может иметь только отдельные особые точки - например, быть конусом, но не объединением двух пересекающихся плоскостей), а алгебраически оно выражается так: кольцо ростков функций в каждой точке должно быть целозамкнуто. Я, конечно, знаю теоремы, которые объясняют, почему это одно и то же, но никакой интуиции, которая говорила бы мне, что так и должно быть, у меня нет. Возможно, просто недостаток воображения.
>> No.161579  
>>161533
Разве про декартово произведение не следует из его определения? То есть, если в одном множестве есть элементы (по условию), и в другом множестве есть элементы (по условию), то можно создать и какую-то пару, с одним элементом из первого множества и одним из второго всилу их непустоты. Нам же не нужно показывать на элементы конкретно.

>>161577
Такое число скорее всего не одно, так что фраза определяет класс чисел.
Должно быть так: "натуральные числа, которые нельзя определить фразой русского языка, содержащей менее чем миллион букв".
>> No.161580  
>>161579
Я, видимо, неточно выразился. Аксиома выбора равносильна не тому, что произведение двух множеств непусто. Она равносильна тому, что произведение произвольного набора множеств непусто.

> Нам же не нужно показывать на элементы конкретно.

То, что нам не нужно указывать на элементы конкретно, и есть аксиома выбора. Она в этом и заключается. Без нее нам именно что нужно указывать элементы конкретно.
>> No.161581  
>>161579
> Должно быть так: "натуральные числа, которые нельзя определить фразой русского языка, содержащей менее чем миллион букв".

Эта фраза все равно противоречива. Фраза "натуральные числа, которые нельзя определить фразой русского языка, содержащей менее чем миллион букв" определяет эти числа, но содержит меньше миллиона букв. Можно добавить слова "наименьшее число", и тогда число будет единственное.
>> No.161585  
https://www.ias.edu/news/2017/vladimir-voevodsky

RIP
>> No.161587  
Воеводский умер.
https://www.ias.edu/news/2017/vladimir-voevodsky
>> No.161590  
А вот, допустим, я захочу углубиться в CS. С чего начать? Чем продолжить? Чтобы шарить в этих ваших когомологиях. Сейчас имеется более-менее полное знание двух семестров дискретной математики (логика высказываний, немного логики предикатов, булевых функций, графов и комбинаторики).

>>161580
Ясно.

>>161581
Да.
>> No.161591  
>>161590
http://mi.mathnet.ru/intf132
>> No.161592  
>>161591
Я прочитал вступление и почти ничего не понял. Так и должно быть?
>> No.161593  
>>161592
Нет, так быть не должно, если ты собрался в когомологии. Прочитай http://mi.mathnet.ru/intf69 тогда.
>> No.161596  
>>161590
> А вот, допустим, я захочу углубиться в CS. С чего начать? Чем продолжить? Чтобы шарить в этих ваших когомологиях.

Какое отношение кс(если, конечно, моя расшифровка этой аббревиатуры как компутер сайенс правильна)имеет к когомологиям?

Энивей, если ты хочешь изучить математику и въехать в когомологии, то тебе придется начать сначала:твой курс "дискретной математики" к настоящей математике отношения не имеет. Нужно будет изучить базовую теорию множеств, общую топологию и основы алгебры. Потом тебе нужно будет изучить мат.анализ и некоторые специальные разделы алгебры: гомологическую, коммутативную и представления групп, хотя бы на уровне владения основными принципами и понятиями. После этого можно будет переходить к алгебраической топологии, науке о гладких многообразиях и комлексному анализу - именно в этих трех науках впервые появляются и используются гомологии, гомотопии и проч. В принципе, после этого этапа уже должна наступать специализация, но если ты предполагаешь изучать геометрию, то дальше должна идти теория групп Ли, а также алгебраическая и дифференциальная геометрия. В какой-то момент нужно будет также выучить базовые теоремы о дифференциальных уравнениях, ибо без них никуда. Конкретизировать эту программу, например, называя конкретные источники по предметам, я не берусь - не настолько самонадеян. Но вполне реальная программа начал математики изложена тут:
http://verbit.ru/Job/HSE/Curriculum/all.txt
>> No.161599  
>>161593
>>161596
Странно, что оба ответивших зацепились за когомологии. Я вообще не знаю что это такое. Вопрос-то был про Компутер Саинс. Упомянул, потому, что это слово было употреблено рядом с гомотопиями (что для меня то же самое) в этой статье >>161585, а они рядом с упоминанием большого вклада в CS. Что такое CS я тоже до сих пор не до конца понял, если бы знал, глупых ответов бы не задавал.
Алсо, кроме дискретки конечно у меня были и есть и другие мат.дисциплины, просто она больше всего похожа на то чтобы являться частью CS.
>> No.161600  
>>161599
> употреблено рядом с гомотопиями (что для меня то же самое) в этой статье >>161585, а они рядом с упоминанием большого вклада в CS. Что такое CS я тоже до сих пор не до конца понял, если бы знал, глупых ответов бы не задавал.

Когомологии к кс не имеют особого отношения. Ты, видимо, не так понял статью - Воеводский преуспел в когомологиях, и внес определенный вклад в кс, но это вещи несвязанные. Твой вопрос был, если сократить, как начать понимать когомологии. Я постарался на него ответить в меру сил. Если ты хочешь понять, что такое кс и как в него вкатиться, то спрашивать нужно, наверное, не тут - cs расшифровывается как computer science, это, грубо говоря, наука посвященная теоретическим вопросам программирования.
>> No.161601  
>>161600
> Если ты хочешь понять, что такое кс и как в него вкатиться, то спрашивать нужно, наверное, не тут
Как же мат логика, на которой работают все компутеры? И вообще, математика — это же язык. Теория типов — это не математика или не CS? И где, если не здесь? В /s/, поди, одни прикладные быдлокодеры, как я,тусуются.
>> No.161602  
>>161601
Ну низнаю. Я сам в кс не сильно разбираюсь. Если здесь есть кто-то, кто хорошо разбирается в сабже - пусть расскажет нам, как дела обстоят реально. Если нет - то тогда тем более он должен спрашивать в другом месте.

> Как же мат логика, на которой работают все компутеры? И вообще, математика — это же язык.

От современной core mathematics, где когомологии и пучки, матлогика таки довольно далека. В особенности далека та матлогика, на которой построены компьютеры(я не имею в виду современную компьютер саенс, в которой и теоркат применение находит, я имею в виду тот теоретический материал, на котором реально строили компьютеры в сороковые-пятидесятые годы) - это именно та "дискретная математика", имеющая мало отношения к настоящей.
>> No.161603  
>>161602
> это именно та "дискретная математика", имеющая мало отношения к настоящей
Извольте! http://mathcenter.spb.ru/nikaan/misc/Two_cultures.pdf
> я не имею в виду современную компьютер саенс, в которой и теоркат применение находит
Он также далёк от когомологий как и от комбинаторики.
>> No.161605  
>>161603
> Извольте!

Извольте что? Я не говорил, что комбинаторика не является "настоящей" математикой. Я говорил, что средний университетский курс "дискретной математики" к математике не имеет никакого отношения. Точно также он не имеет отношения к сложной комбинаторике, вроде описанной в статье, ссылку на которую ты кинул.

> Он также далёк от когомологий как и от комбинаторики.

Ну и?
>> No.161606  
>>161603
> Извольте!

Извольте что? Я не говорил, что комбинаторика не является "настоящей" математикой. Я говорил, что средний университетский курс "дискретной математики" к математике не имеет никакого отношения. Точно также он не имеет отношения к сложной комбинаторике, вроде описанной в статье, ссылку на которую ты кинул.

> Он также далёк от когомологий как и от комбинаторики.

Ну и?
>> No.161607  
>>161605
> Я говорил, что средний университетский курс "дискретной математики" к математике не имеет никакого отношения
Средний университетский курс «математического анализа» тоже имеет с математикой мало общего.
>> No.161627  
Как доказать что функция f(x, y) = (y−x^2)(y−3x^2) не имеет экстремума в (0,0), но любое ее straightening to straight проходящей через (0,0) имеет сильный минимум в 0?
>> No.161629  
1507037116499.png (0.0 KB, -1x-1)
0
Будит ли выпуклая оболочка замкнутого множества замкнутой?
>> No.161630  
>>161627
> straightening to straight

Что это, блядь, означает? Это попытка выебнуться? "ограничение на прямую" по-английски будет restriction to a line.

Доказать элементарно - проходящая через (0,0) прямая имеет уравнение y=ax(если прямая не вертикальна, конечно). При подстановке в функцию получаем а^2 х^2 - 4а х^3 + 3х^4. У этой функции производная в нуле нулевая а вторая производная равна 2а^2. Это значит, что вторая производная положительна при а =!=0, что автоматически дает минимум. При а=0 достаточно подставить y=0 в уравнение. Осталась прямая x = 0, но это тоже проверяется элементарно.
>> No.161631  
>>161630
Это значит что я не знал как по-русски будет "ограничение на прямую", спасибо, коть.
>> No.161632  
>>161631
Так и по-английски не знал же, не?
>> No.161703  
Анон, последовательность это последовательность, а ряд - сумма всех ее членов?
>> No.161706  
>>161629
Нет. Контрпример - интервал.
>> No.161707  
>>161706
К чему это контрпример? Интервал не замкнут. Отрезок замкнут, и является своей собственной выпуклой оболочкой.
>> No.161708  
>>161703
Ну да, примерно так.
>> No.161710  
Существуют ли ассоциативные (конечные) квазигруппы, не являющиеся лупами ?
>> No.161711  
>>161710
Для любого a, есть такие x и y, что

a x = a
y a = a

Домножая и используя ассоциативность, получаем:

a (x y) = a y ⇒ x y = y
(x y) a = x a ⇒ x y = x

Отсюда x = y, т.е для любого a левая и правая единицы совподают.

Равенство единиц для любых двух a и b доказывается элементарно.
>> No.161712  
>>161711
Зачем вы это делаете на доброчане? Есть же специальные пасто-ориентированные держатели текстов для формул, удобные.
>> No.161713  
>>161712
Ну если ты наставиваешь...
http://mathb.in/19178
>> No.161775  
Простые-числа.png (0.0 KB, -1x-1)
0
Причём здесь свойства делимости суммы, объясните дураку?
>> No.161776  
>>161775
Гугл находит мне это:
> Свойство 2. Если каждое слагаемое суммы, кроме одного, делится на некоторое число, а одно не делится, то и вся сумма не делится на это число.
Вообще, вряд ли математики выражаются подобным образом.
>> No.161777  
Что больше, гипотенуза или сумма катетов?
>> No.161778  
>>161777
Сумма катетов.

При положительных a и b
a^2 + b^2 + 2ab > a^2 + b^2, т.е. a+b > sqrt(a^2 + b^2).
Если a и b - длины катетов, то sqrt(a^2 + b^2) - гипотенуза.
>> No.161784  
>>161711
Спасибо
>> No.161785  
>>161784
Доброчане слишком добрые [неблагородные]. Почему не дать ему ссылок/не описать для него, что есть теорема Пифагора и какие свойства у алгебраических выражений, в т.ч. равенство, при возведении в степень/под корень?
Это же не вопрос-ответ сервис, а комьюнити, так почему не дидактика а вместо этого приводить просто доказательство без пояснений особо (арифметика чистая)?
>> No.161787  
>>161785
Комьюнити тут так себе, на мой вкус.

Что до дидактики, то меня она всегда только раздражала, я всегда предпочитал увидеть доказательство конкретного факта, а не слушать намеки. Того же и желаю другим.
Энивей, никто не мешал тебе написать ему ответ в твоем вкусе. Но ты этого не сделал.
>> No.161789  
>>161785
> Почему не дать ему ссылок/не описать для него, что есть теорема Пифагора и какие свойства у алгебраических выражений, в т.ч. равенство, при возведении в степень/под корень?
Это оставлено читателю в качестве упражнения.
>> No.161796  
Посоветуйте годный учебник для совсем далёкого от математики человека
>> No.161799  
>>161796
Годный учебник чего и для чего? Одно дело - для подготовки к чему-либо, другое - просто из интереса. И какой у тебя уровень сейчас? Школоматематику помнишь?
>> No.161800  
>>161799
> какая
Общая чтобы потом стать математиком
> помнишь ?
Немного помню школьную,немного помню начало анализа
>> No.161802  
>>161800
Начать можно с теории множеств, общей топологии и алгебры. По теории множеств я ничего советовать не берусь, меня самого эта тема никогда не интересовала, топология - Вербицкий, Фукс-Фоменко. Алгебра - Ван дер Варден, Ленг. По анализу можно почитать Зорича, Лорана и "элементарную теорию аналитических функций" Картана.
>> No.161804  
>>161802
Спасибо
>> No.161915  
Как отличаются доказательства "тогда" и "тогда и только тогда"? Какой уникальный признак второго?
>> No.161916  
>>161915
"Люди смертны" - утверждение типа "тогда". Я человек, следовательно я смертен, но в обратную сторону оно не работает - моя собака тоже смертна, но она не человек.

Утверждения типа "тогда и только тогда" работают в обе стороны: число делится на три тогда и только тогда, когда сумма его цифр делится на три. Это означает, что если число делится на три, то сумма его цифр делится на три, и если сумма цифр числа делится на три, то число делится на три.
>> No.161917  
>>161915
В дополнение к анону, скажу, что "тогда" - это тоже самое что "из А следует В", а "тогда и только тогда" - это "из А следует В и одновременно из В следует А". Утверждения второго типа еще называют "критерий". Например критерий сходимости.
>> No.161918  
>>161916
>>161917
А как бы вы тогда описали разницу в доказательстве?
>> No.161919  
>>161918
В одном случае тебе надо доказать, что:
> "из А следует В"

А в другом две вещи:
> "из А следует В"
> "из B следует A"
>> No.161920  
>>161919
Хорошо. Ну примерно так я и представлял.
>> No.161921  
Я правильно понял суть?

Докажите, что множество A тогда и только тогда является подмножеством множества B, когда каждый элемент, не принадлежащий B, не принадлежит A.

1. Множество A тогда и только тогда является подмножеством множества B, когда - каждый элемент, не принадлежащий B, не принадлежит A.
2. Каждый элемент, не принадлежащий B, не принадлежит A, тогда и только тогда, когда - множество A является подмножеством множества B.

1а. Предположим что А - не подмножество В. Тогда элемент х принадлежащий В, не принадлежит и А.
2а. Предположим, что элемент х принадлежит А и не принадлежит В. Тогда А не принадлежит В.
>> No.161922  
>>161921
1а. Предположим что А - не подмножество В. Тогда элемент х, принадлежащий А, не принадлежит В.
2а. Предположим, что элемент х принадлежит А и не принадлежит В. Тогда А не принадлежит В.
Сам исправил.
>> No.161923  
>>161921
В 1а написана какая-то бессмыслица. Элемент, принадлежащий В, может не принадлежать А даже когда А - подмножество В.
>> No.161924  
>>161922
Так примерно правильно, хотя я бы добавил пару кванторов( существует х, принадлежащий А и не принадлежащий В).
>> No.161931  
Если поверхность задана как F(x,y,z)=const, то градиент F в точке (x0,y0,z0) есть вектор нормали к этой поверхности в этой точке?
>> No.161932  
>>161931
Если только он ненулевой. Легко построить поверхность с перегибом, у которой градиент будет зануляться, а нормаль вполне будет.
>> No.161976  
А мне стало интересно, когда впервые появились доказательства "тогда и только тогда"?
>> No.161977  
>>161976
В доказательствах "тогда и только тогда" нет ничего особенного, качественно отличающего их от других доказательств. Это просто доказательство "тогда", проделанное дважды - в одну и в другую сторону. Поэтому ответ на твой вопрос - тогда же, когда и все остальные математические доказательства.
>> No.161994  
-581.jpg (0.0 KB, -1x-1)
0
Как доказать что если функция выпуклая, то эта функция в квадрате тоже выпуклая?
>> No.161995  
>>161994
Все даже печальней. Квадрат функции это скорее f(x) * f(x) или f(f(x))?
>> No.161996  
>>161995
Квадрат синуса это sin(x)*sin(x) или sin(sin(x))?
>> No.161997  
>>161996
sin(x)*sin(x)? Я умничка?
>> No.161998  
>>161997
Ок. Тогда квадрат это f(x) * f(x).
>> No.162000  
>>161994
Никак. Если функция принимает отрицательные значения, то ее квадрат будет менять направление выпуклости.
>> No.162001  
>>162000
Но она останется выпуклой?
Я попытался решить от обратного через определение выпуклости и если предположить что f^2(x) не выпуклая, то получается (f(x1) - f(x2))^2 < 0, чего не может быть?
>> No.162004  
Чем отличается аксиома выбора от аксиомы детерминированности?
>> No.162053  
-709.jpg (0.0 KB, -1x-1)
0
Анон, конструкцию типа
y'' + f(x)y' + g(x)y = h(x)
всегда можно свести к конструкции
y'' + v(x)y = w(x)
или не всегда? Если можно, то как это в общем виде сделать?
>> No.162093  
Посоветуйте книжечку или набор статей про начальный теорвер.
>> No.162097  
1-24.png (0.0 KB, -1x-1)
0
Что делать если я случайно пикрилейтед? С чего начать хотя бы?
>> No.162098  
>>162097
У тебя там некоторых определений не хватает. Например, про функцию Ф говорят, что она возрастающая. Но Ф - функция двух переменных, а возрастание и убывание для таких функцию всюду определяется по своему.

Ну, разберу, например, третью задачу. Нужно доказать, что, что Z - замкнутое мн-во, т.е. что если zi - сходящаяся последовательность, и все zi лежат в Z, то и z = lim zi лежит в Z. Это сразу следует из определения полунепрерывности снизу: если все zi лежат в Z, то f(zi) =< lambda для всех i. Дальше, применив определение полунепрерывности снизу для посоедовательности zi, получим, что f(z) =< lambda, т.е. z лежит в Z.
>> No.162099  
>>162093
Гмурман В.Е Теория вероятностей и математическая статистика.
>> No.162100  
>>162053
Если уравнение однородное(т.е. справа 0), то свести можно всегда - см, например, тут: http://mathworld.wolfram.com/Second-OrderOrdinaryDifferentialEquation.html

Неоднородное уравнение сводится к однородному, если знать хоть одно частное решение неоднородного - если z - частное решение, то при замене y на y-z получается однородное уравнение.
>> No.162101  
1507204493162123033.png (0.0 KB, -1x-1)
0
>>162100
Мм, спасибо большое за ответ. Ясно теперь.
>> No.162133  
Бля, ононы, я тут осознал ведь логарифм - это просто зеркальное отображение экспоненты. Есть какая-нибудь математика без ебучих иксов, игриков, функций этих ебучих, а чисто про линии, кубики, шарики? Ну как школьная геометрия, только круче.
>> No.162134  
>>162133
Выучи матанализ. Он про это.
>> No.162135  
>>162133
> Бля, ононы, я тут осознал ведь логарифм - это просто зеркальное отображение экспоненты
Лол.
> Есть какая-нибудь математика без ебучих иксов, игриков, функций этих ебучих
Нет.
> про линии, кубики, шарики
Да.
>> No.162137  
>>162133
> Есть какая-нибудь математика без ебучих иксов, игриков, функций этих ебучих, а чисто про линии, кубики, шарики?
Топология, например.
>> No.162144  
>>162137
Это про шарики. Про кубики — алгебраическая геометрия.
>> No.162153  
Объясни, математик-кун, что в твоей любимой науке особенного? Только объясни, закрепляя жизненным опытом, своим личным, а не опираясь на вычисления расстояния до другой планеты и подобного. Объясни в каких моментах логики людям может помочь курс школьной математики - это закрепляется подсоздательно, люди строят в голове цепочки из треугольников, уравнений, как? Откуда эти восторженные статьи про величество математики, но всё - самовнушение без нормального подкрепления фактами, как именно повлияла математика, кроме работы мозгов на уроках (с таким же успехом можно собирать пазлы, рисовать, писать стихи, я не прав?). Выглядит, будто человек пытается оправдать свою пустую трату времени чужими достижениями. Приводит ум в порядок, но на каком уровне? Если же приводит, то наверняка нужен правильный подход ко всем этим учебникам величайшей науки. Что это за подход?
>> No.162155  
>>162153
В школьной математике действительно нет ничего примечательного.
>> No.162156  
>>162155
Можешь что-нибудь порекомендовать, чтобы выжать хоть какой-то профит? Уже убедился, что преподавание отстой, лучше самому изучать и решать, но зачем тогда все эти темы? Ради экзамена?
>> No.162157  
>>162156
Вот некоторые идеи, которые должен доносить курс математики:

- Могущество простых, но могущественных идей
- Не все комбинации слов осмысленны
- Идея доказательста (не буква, но дух!)
- Выбор языка имеет значение
- Суть, однако, не зависит от языка
- Иногда задача это другая задача
- Иногда задача лишь часть задачи
- Бывают невозможные вещи

Люди слишком много говорят о том, как использовать математику, и слишком мало — о том, как её создавать. Точнее: про то, как по другому взглянуть на проблему, над которой работаешь.

Какую проблему? Я не знаю. Может, ты ваяешь искусство из кода. Может, делаешь что-то практичное, а может — просто играешь с идеями. Не важно. Математика это способ мышления.
>> No.162166  
111-5.png (0.0 KB, -1x-1)
0
Помогите решить эту задачу так чтобы я мог напить функцию на питоне (методом Эйлера). a, b, k - положительные константы, вводятся при запуске.
>> No.162168  
>>162157
Это всё логика же. Ну, можно использовать математику, но... По-моему, многие люди так не воспринимают, просто решают по формулам, находят значения. Это увлекательно, тренирует мозги?
>> No.162169  
>>162166
Ты и прямо сейчас, сходу, можешь её написать.
>> No.162170  
>>162169
Написал, даже несколько таких. Теперь стыдно.
>> No.162172  
>>162153
Я хоть ни разу и не математик, но постараюсь ответить. Математика развивает абстрактное мышление. Другими словами, она учит строить абстракции общие настолько, насколько это имеет смысл, но не больше. Вот эта "но не больше" часть очень важна, это целое искусство, которое развивается параллельно математическому чутью. Чем абстракция более общая, тем она мощнее (т.к. охватывает больше сущностией), но с другой же стороны бесполезнее (т.к. охватывает больше сущностей). Примеры хороших, годных абстракций: сопряженные функторы, модули над кольцом, цепные комплексы, схемы. Нахрена это нужно? Да потому что это безумно красиво. Это как наркомания, только бесплатно: я могу обосранным лежать в луже мочи и блевотины где-нибудь в коллекторе теплотрассы, но размышление о гомотопических группах сфер будет делать меня по-настоящему счастливым. В общем, смотри лекции Ромы Михайлова https://youtu.be/hQLxmSgL0yI и читай Алуффи https://amazon.com/dp/0821847813.
>> No.162173  
Доброаноны, посоветуйте учебник по мат. анализу. Пробовал Зорича и Рудина. Сразу видно, крутые учебники, но меня огорчает, что все задачи - доказательные, и к ним нет ответов. Я вообще не понимаю, как что-либо доказать или опровергнуть, а если какие-то упражнения и смог решить, то не знаю, правильно ли.
Или лучше пойти на курсы от MIT OCW и Coursera?
В общем мне нужен учебный материал с неустаревшими на сегодняшний день знаниями, с упражнениями и, самое главное, ответами на упражнения.
>> No.162174  
>>162173
https://www.khanacademy.org/math/calculus-home
>> No.162175  
>>162174
Забыл о существовании этого проекта. Как курс, по сравнению с MIT и Coursera?
>> No.162176  
>>162175
Не знаю, мне не пришлось повторять еще и на MIT и Coursera.
>> No.162177  
>>162168
> По-моему, многие люди так не воспринимают, просто решают по формулам, находят значения.
Да, но это не проблема конкретно математики.
> Это увлекательно, тренирует мозги?
Нет. Но это и не математика.
>> No.162178  
>>162173
Анон, ты пытаешься бороться с симптомами болезни вместо того, чтобы устранить, собственно, причины. Тебе не учебник по анализу нужен – рано ещё, а хорошая книжка по школьной алгебре, класса так с шестого. Пока ты сам не научишься доказывать теоремы, ничего путного из твоих занятий математикой не выйдет. Это сложно принять, еще сложнее исправлять, но не обманывай хотя бы себя. Возьми Алгебру Гельфанда и начинай с азов. Как окрепнешь, станешь более увереннее в себе, берись за олимпиадные задачки, так же начиная от для самых маленьких. Математика это все-таки про смекалку, а не обезьяньи 1000 однотипных примеров на вычисление производных и первообразных по алгоритму из книжки.
>> No.162186  
>>162178
Хмм, но это для чистых математиков, ведь так? Для прикладных задач такое разве нужно?
>> No.162187  
>>162186
Ты хочешь научиться доказывать теоремы? Ты думаешь, прикладные математики их как-то по-иному доказывают?

Так-то, можно вообще ничему не учиться, решать все по формулам, которых сам не понимаешь. Но если хочешь что-то доказывать - другого пути нет.
>> No.162188  
>>162187
Спасибо за Гельфанда, как раз мне подходит.
В школе я нормально решал математику, в универе похуже, но у меня не сложилось какого-то системного взгляда. То есть я не могу описать какую-то проблему языком математики, все эти формулы остались просто формулами, без какой-то связи друг с другом и реальным миром.
>> No.162189  
Давным-давно слышал про некую программу, которая помогает математикам, при помощи вычислительной мощности твоей системы, доказать теорему какую-то/найти хитровыебанное число/или что-то подобное. Вроде связано с теорией чисел (не точно). Может кто напомнить конкретнее.
>> No.162190  
>>162189
gcc
>> No.162194  
>>162189
См:
https://en.wikipedia.org/wiki/Proof_assistant
https://en.wikipedia.org/wiki/Satisfiabilitymodulotheories
https://www.youtube.com/watch?v=gcvl08ghh3s
>> No.162208  
В обычном 3-х мерном кубе угол каждой плоскости соприкасается с 2 другими плоскостями. Как узнать, с каким количеством плоскостей будет соприкасаться угол одной плоскости 4-х и более -мерного куба?

c: зомг быстрее
>> No.162209  
>>162208
Представь куб, заданный (гипер)плоскостями:
x_1 = ±1
x_2 = ±1
...
Пусть угол задан плоскостями x 1 = 1, x 2 = 1. Тогда уравнения x_3 = ±1, ... задают оставшиеся плоскости.

Получается 2 (n - 2), где n — размерность пространства. 0 для 2D (ни одна прямая не пересекает оба луча одновременно), 2 для 3D, 4 для 4D и так далее.
>> No.162212  
>>162209
Что-то у тебя в вычислениях не то. Почитал, посчитал, получилось, что в каждой вершине 4-х мерного куба сходится 6 двумерных граней.
>> No.162216  
>>162212
Я посчитал гиперплоскости же. Обычные плоскости считаются немного сложнее.
Но идея всё та же: сопоставить каждой плоскости уравнение и посчитать.
>> No.162220  
>>162216
Аа, понял тебя. Я неправильно вопрос сформулировал.
>> No.162226  
Есть ли какие-то теоремы, утверждающие, что неопределенный интеграл от какой-то функции нельзя выразить в элементарных функциях? Или это больше 'чтош, никто не смог взять этот интеграл за 200, назовем неберущимся и хуй с ним'?
>> No.162227  
>>162226
https://ru.wikipedia.org/wiki/Алгоритм_Риша
>> No.162230  
Вопрос по простейшим ЧМам.
Вот естьу нас метод биссекции вычисления собственных значений симметричной матрицы NxN. Он основан на том, что мы знаем, что все собственные значения лежат в неком промежутке, который по матрице можно вычислить за один проход и память N и теоремы о том, что количество отрицательных собственых значений на промежутке равно количеству смен знака в последовательности миноров. Тогда простым делением отрезка пополам мы с некой точностью находим все собственные значения. Для этого мы приводим каким-нибудь методом матрицу к трехдиагональной, а затем применяем метод биссекции. Например методом домножения на матрицы вращения. Вопрос, почему нельзя привести матрицу к просто диагональному виду?
>> No.162231  
>>162230
> Например методом домножения на матрицы вращения. Вопрос, почему нельзя привести матрицу к просто диагональному виду?

Так ведь не все матрицы приводятся к диагональному виду же. Возьми в трехмерном пространстве какое-нибудь вращение с растяжением по оси вращения - у него и не будет диагонального вида. А к трехдиагональному - все.
>> No.162242  
2l77XrJSf4Y.jpg (0.0 KB, -1x-1)
0
Анон, реши мне домашку
Пропустил несколько дохуя пар, как решать такое?
Насколько я понял эта тема называется 'интеграл с параметром', но я в душе не ебу, чем они отличаются от вычисления обычных интегралов.
>> No.162243  
>>162242
По параметру можно дифференцировать (если выполнен ряд условий), вычислить более лёгкий интеграл, а потом полученное значение обратно интегрировать по параметру.

В некоторых примерах нужно использовать уже вычисленные человечеством интегралы.
>> No.162246  
Анон, что там есть по взятию обычных интегралов? Я знаю про подстановки Эйлера и универ. триг. подстановку, но я помню, что их есть ещё дофига разных.
>> No.162352  
расчеты.png (0.0 KB, -1x-1)
0
Аноны мне кажется я неправильно употребляю знак принадлежности в данном случае. Здесь должно быть условие существования X штрих на основании пределов справа
>> No.162355  
>>162352
Мне
а)совершенно непонятно, что ты там рассчитываешь.
б)абсолютно не хочется вглядываться в пейнтовские каракули.

Думаю, остальным анонам тоже. Если хочешь содержательного ответа, оформи вопрос по-человечески.
>> No.162360  
>>162355
а) не важно что я там рассчитываю , я просто спрашиваю можно ли использовать знак принадлежности который я выделил в конце при обозначениии сушествования X штрих (который там же в конце) на множествах справа от знака принадлежности.
б) в принципе мне уже не важно , это все равно для моего личного пользования
>> No.162378  
>>162242
Это нельзя решать, это превратит тебя в интеграста. Скажи преподу, что он застрял в 30-х годах прошлого века, и кинь в него картофаном.
>> No.162414  
Можно вопрос от мимонуба? Читаю тут про квантовые языки программирования (QPL, QCL, QML, Q#).
Получается, программист будущего будет в большей степени физиком, нежели математиком? Готовящимся стать программистами уже сейчас нужно дополнительно налегать на физику? Придётся потом долго переучиваться?
>> No.162415  
>>162414
Нет, налегать на физику никакого смысла нет. Потому что программирование никогда не было связано с физикой, даже во времена машинных кодов. Аналогично для работы с TensorFlow совершенно не нужно и даже вредно читать учебник Ленга по алгебре. Все необходимые понятия для работы с "квантовыми языками" толковый программист схватит очень быстро. Они по абстрактности никак не превосходят паттернов ООП, а паттерны изучаются почти мгновенно, же.
>> No.162417  
>>162415
То есть, базой в любом случае останется математика, я правильно понял? Спасибо за пояснения.

Кто-нибудь пробовал экспериментировать с этими языками? Это вообще возможно делать на обычном ПК или нужно арендовать мощности у IBM и подобных?
>> No.162420  
>>162415
Некорректное сравнение. Это как советовать не заморачиваться над пониманием принципа работы вычислительной техники в шестидесятых годах, потому что через 50 лет можно будет просто подключить миллиард уже готовых библиотек для фортрана, не сильно уже заморачиваясь над тем, как там всё внутри работает. Рим не сразу строился, то есть либы для фортрана не за один день написали. А уж сколько времени и человекочасов на проработку базы, необходимой для реализации TensorFlow, ушло - это вообще с ума можно сойти.
>> No.162421  
>>162420
В шестидесятых, на минуточку, уже существовали бейсик и алгол. Ты утверждаешь, что необходимо изучать уравнения Максвелла, чтобы писать программы на бейсике?
>> No.162425  
>>162421
Я утверждаю, что без понимания основ всё, на что ты будешь способен - это бездумное использование уже готовых моделей и работа с ними, как с чёрными ящиками. И ты не один будешь таким умным, поэтому придётся конкурировать с другими обезьянами, которые не хуже тебя научатся нажимать на кнопки по очереди. Чем глубже понимание, тем меньше конкуренция.

Уравнения Максвелла или Шрёдингера никто понимать не заставляет, однако неразумно отрицать то, что их понимание немного расширяет перспективы для гипотетического специалиста.
>> No.162427  
>>162425
Но вообще сравнение неправильное опять же. Бейсик - это прикладной уровень. Сейчас, например, совершенно не обязательно уже знать, что такое CISC/RISC и чем они друг от друга отличаются. Не нужно заморачиваться с пониманием принципов трансляции, потому что компилятор всё сделает за тебя и на порядок эффективней. Теперь и вычислительных мощностей столько, что можно тысячи абстракций друг на друга наворотить для вывода обычного Hello World, и такая программа отработает за разумное время. Но даже сейчас между реализацией алгоритма за O(2^n) и за O(n log n) огромная разница.
>> No.162428  
>>162425
> никто понимать не заставляет
То есть программисту совершенно не обязательно знать физику, чтобы программировать. Ведь даже такие азы, как уравнения Максвелла, современному программисту нафиг не нужны.
> которые не хуже тебя научатся нажимать на кнопки
Программирование - это не про "нажимать на кнопки", вообще говоря. В программировании очень много своих собственных заморочек. Изучение одной какой-нибудь либы для крестов повышает уровень человека как программиста гораздо существеннее, чем изучение пусть даже всего курса общей физики.
>> No.162450  
Можно ещё один глупый вопрос? Допустим, есть маршрутизатор который поддерживает ipv6 и только его. Должна ли внешняя, подключаемая по usb сетевая карта с ipv4 поддерживать ipv6 или они обратно совместимы ? А если представить сеть, в которой уже только ipv6, воткнуть просто провод, а другим концом в старый комп, будет нормально работать?
>> No.162451  
>>162450
Иди в /lor.
>> No.162453  
Что нужно чтобы вкатиться в так называемую "Высшую математику"? Квадратное уравнение решить могу, что такое логарифм знаю, с геометрией всё просто ужасно, не мыслю в системах уравнений. Что читать и решать? Что-то, где больше практики, меньше воды и теории, желательно, полагаю. Хотелось бы ещё обсудить с кем-то эту тему, поэтому взываю математиков из этого треда.
>> No.162454  
>>162453
Давай сразу уточним. В "высшую", как её преподают в инженерных вузиках? Или в чистую математику, с категориями и гомотопическим хаосом?
>> No.162456  
>>162454
Вот как в вузиках с техническими направлениями. Чтобы оттуда вытащить весь нужный базис и дальше уже в конкретную область математики я сам буду пробовать.
>> No.162457  
>>162456
Тогда читай сразу Демидовича, "Краткий курс высшей математики". Дополнительные книги для него не нужны.
>> No.162459  
>>162457
То есть школьный курс(после 9го ушёл в фастфуд работать) по математике вообще не нужно учить? С тригонометрией знаком и некоторыми темами тоже. Просто хочу для себя сразу уточнить, нужно ли мне будет открывать или изучать ещё что-то(та же система уравнений). Ибо говорят, что в математике все темы очень сильно связаны. Вот примерно это последний вопрос, который я хотел задать.
>> No.162480  
>>162459
Начнёшь читать Демидовича - поймёшь, нужно ли лично тебе обращаться к школьным учебникам. Скорее всего, не нужно.
>> No.162483  
>>162454
> чистую математику, с категориями и гомотопическим хаосом
Можешь объяснить, что это, и почему матан не преподают так в моем техническом ВУЗе?

Ещё хотел спросить, а что была за прикольная штука о выведении всего из нуля? В этом или одном из предыдущих тредов постили. Что-то вроде определим пустое множество, пару действий с множествами, и подроча ими, получим все различные множества чисел.
>> No.162497  
>>162483
Это современные математические объекты, важные так же, как множества сто лет назад. Математика - это живая исследовательская наука, и в ней постоянно появляются клёвые штуки. В России есть три места, где готовят математиков-исследователей. Это матфак ВШЭ, мехмат МГУ и матмех СПбГУ (в порядке убывания крутизны), в этих местах можно столкнуться с гомологиями. В остальных местах нельзя, потому что в них математика отсталая. Считается, что инженерам не нужна чистая математика, поэтому матан им преподают по рабоче-крестьянски.

То, о чём ты говоришь, называется кумулятивный универсум фон Неймана. Гуглится.
>> No.162499  
>>162497
> инженерам не нужна чистая математика

А ты можешь сказать, кому нужна чистая математика? ответ 'чистым математикам' не принимается
Многие часто говорят, что математика это инструмент для других дисциплин типа физики, химии и прочего.

За универсум спасибо.
>> No.162503  
>>162499
Математика - это самостоятельная наука, и она не обязана быть нужной кому-то, кроме самих учёных. Как и любая другая фундаментальная наука, собственно. Мне кажется, что науки существуют ради поиска истины. Мне не симпатична позиция, что науки нужны постольку, поскольку увеличивают надои чугуна.
>> No.162517  
Screenshot_7.png (0.0 KB, -1x-1)
0
Быть может, кому известен софт, который позволяет максимально просто вывести формулу на основе многомерных значений.

Пытаюсь голыми руками вывести формулу, зависящую от 4 переменных, на основании наборов значений, но процесс идёт как-то не очень продуктивно.
>> No.162518  
>>162517
Вольфрам математика, ListInterpolation.
http://reference.wolfram.com/language/ref/ListInterpolation.html
>> No.162519  
>>162518
Спасибо.
>> No.162545  
1-16.jpg (0.0 KB, -1x-1)
0
>>160159
Анон, мне кажется я ебанулся, помоги с формулой Тейлора. Есть 3 вопроса.
1) Смотрим на определение ряда. Видим, что о-малое будет такой же степени, что и та, до которой разложили икс. Разложили до 3, будет третья, до 5, будет пятая. Тогда почему у синуса и у косинуса в примерах остатки написаны степени на 1 больше, чем степень икса? Вот я разложил синус и у него должно быть 2n-1, а не 2n. Что я не понимаю?
2)Перечитал о-символику и вспомнил, что мы пишем что-то типа f(x)=o(g(x)) ПРИ x->a. Вот в разложении Тейлора остаток о(x^n) , он о-малая от х^n при стремлении куда? К точке х0 в которой раскладываем?
3)Сколько членов разложения в точке 0 того же синуса мне нужна взять, чтобы этот многочлен был эквивалентен синусу в 2? в 5? в 100? на бесконечности?
>> No.162548  
Screenshot-from-2018-01-13-13-38-06.png (0.0 KB, -1x-1)
0
>>160159
Готовлюсь к ЕГЭ, зазубриваю формулы до автоматизма, т.к. в 17 году набрал всего 72 балла, что для лба 26 лет как-то несолидно.
Так мало набрал именно потому долго вспоминал/выводил очевидное.

Тем не менее, сидя на одном из любимых сайтов по математики наткнулся на странную формулу площади треугольника (пикрилейтед).

Кто-нибудь может подсказать про ее вывод? А то я совсем не шарю зачем столько минус первых степеней, и почему она так похожа на формулу Герона). А не понимая формулы (хотя бы приблизительно) я ее выучить не могу.
>> No.162557  
>>162545
1)Потому что в ряде Тейлора синуса(соотв., косинуса) только нечетные(соотв., четные степени), и в добавочном члене мы, таким образом, через степень перескакиваем.
2)Да.
3)Что ты подразумеваешь под эквивалентностью?
>> No.162576  
>>162548
`Пусть a, b, c - длины сторон.
2S = a ha = b hb = c hc
Отсюда a = 2S/ha, b = 2S/hb, c = 2S/hc.

Полупериметр:
p =
= (a+b+c)/2
= (2S/ha + 2S/hb + 2S/hc)/2
= 2S(1/ha + 1/hb + 1/hc)/2
= 2SH

Заметим, что
2SH-a = 2SH-2S/ha = 2S(H-1/ha).
Аналогично для b и c.

По формуле Герона,
S^2 =
= p(p-a)(p-b)(p-c)
= 2SH(2SH-a)(2SH-b)(2SH-c)
= 2SH 2S(H-1/ha)2S(H-1/hb)2S(H-1/hc)
= 16S^4 H(H-1/ha)(H-1/hb)(H-1/hc)

Т.е. 1 = 16S^2 H(H-1/ha)(H-1/hb)(H-1/hc).

Значит,
S^2 = 1/(16 H(H-1/ha)(H-1/hb)(H-1/hc))
S = 1/(4 sqrt(H(H-1/ha)(H-1/hb)(H-1/hc)))
`
>> No.162577  
>>162576
Спасибо за пояснение, анон.
>> No.162639  
1516296796349.png (0.0 KB, -1x-1)
0
Анон, я полный нуб в этой теме, посоветуй что почитать, чтобы я на выходе понимал как сделать следующее:
Есть тор, координаты на котором описываются как элемент [0,1)x[0,1)x...x[0,1). На нем разбросаны n точек. Я хочу найти многочлен (на плоскости степени n-1, тут видимо тоже), который проходил бы через все точки.
>> No.162642  
>>162639
Что значит "проходил бы"? Многочлен никуда не идет и ни через что не проходит.

Тебе нужно найти многочлен с нулями в этих точках? Параметризованную многочленом кривую, проходящую через эти точки?
>> No.162647  
>>162642
> Параметризованную многочленом кривую, проходящую через эти точки?
Это.
Ну и вообще, понять как работает (вычислительная!) геометрия во всяких сомнительных непрерывных пространствах.
>> No.162648  
>>162647
Реши задачу для одномерного тора(кстати, тор не является произведением интервалов). В этом тебе поможет интерполяционный многочлен Лагранжа. Потом пойми, что задача в общем случае имеет то же самое решение - параметризация кривой на декартовом произведении есть декартово произведение параметризаций.
>> No.162649  
>>162648
> кстати, тор не является произведением интервалов
Да, я догадываюсь.
> для одномерного тора
ээ.. Для окружности? Там как бы одна такая кривая.
Алсо, я понимаю, что ты хочешь натолкнуть меня на решение, но мне не принципиально решение конкретно этой задачи, я хочу книжку с общими принципами. Что-то про многообразия, наверное? Главное, чтобы была не только чистая абстрактная топология, но в первую очередь вещи, которые можно взять и посчитать в скрипте на питоне, например. В координатах.
>> No.162650  
15159625379551.png (0.0 KB, -1x-1)
0
Сяпки анончики.
Готовлюсь к ЕГЭ, дальнейшая цель связать свою жизнь с математикой. Нужен автор более менее годных учебников по школьному курсу алгебры
>> No.162651  
>>162650
В дополнение, мой сегодняшний уровень знаний 8 класс, по учебникам макарычева.
>> No.162652  
>>162649

> > для одномерного тора
> ээ.. Для окружности? Там как бы одна такая кривая.


Там бесконечно много таких кривых, поскольку кривая может проходить через точки при разных значениях параметра. И да, задачу можно решать не на окружности, а на отрезке, а потом отждествить точки 0 и 1.

> Алсо, я понимаю, что ты хочешь натолкнуть меня на решение, но мне не принципиально решение конкретно этой задачи, я хочу книжку с общими принципами.

Видишь ли, конкретная задача, про которую ты спрашиваешь, плюс-минус тривиальна. Ну, простая она. Тут нет никаких общих принципов, исходя из которых можно было б рекомендовать книжки. Обозначь более широкий класс задач, я что-нибудь придумаю.
>> No.162654  
>>162652
> Обозначь более широкий класс задач, я что-нибудь придумаю.
Матан и аналитическая геометрия на кривых поверхностях.
> Там бесконечно много таких кривых
Нет, там ровно одна такая гладкая замкнутая кривая, совпадающая с самим пространством. Какие, к хуям, параметры?
> Ну, простая она.
Да, я могу развернуть тор в тот же квадрат, интерполировать на плоскости, свернуть обратно и что-то получить. А если я хочу, например, минимизировать площадь, которую ограничивает эта кривая? Будет ли оно работать так же, как на плоскости? Или если метрика как-то хитро связана с поверхностью?
>> No.162656  
>>162654
> Матан и аналитическая геометрия на кривых поверхностях.

Дифференциальная геометрия, гладкие многообразия. Начни с анализа на многообразиях, например, в учебнике Зорича или Львовского, потом дифференциальная геометрия(например, одноименная книжка Шварца или Номидзу).

> Какие, к хуям, параметры?

Ты что, думаешь, существует единственное отображение из отрезка на отрезок?

> > Ну, простая она.
> Да, я могу развернуть тор в тот же квадрат, интерполировать на плоскости, свернуть обратно и что-то получить.


Ну да.

> А если я хочу, например, минимизировать площадь, которую ограничивает эта кривая? Будет ли оно работать так же, как на плоскости? Или если метрика как-то хитро связана с поверхностью?

Это уже очень сложные вопросы, причем совершенно отличные от исходного. Даже не знаю, где подобное можно искать. Кажется, чем-то похожим занимался М.Громов, можно поискать по словам "метрическая геометрия".
>> No.162658  
>>162656
> > А если я хочу, например, минимизировать площадь, которую ограничивает эта кривая?
Вдогонку, я, честно говоря, не уверен, что эта задача решена даже для плоскости. Я даже не уверен, что для произвольного набора точек вообще существует замкнутая алгебраическая кривая, проходящая через них.
>> No.162664  
>>162658
> что эта задача решена даже для плоскости
В смысле, что площадь не у любой фигуры определена? Ну да, не у каждой.
> для произвольного набора точек вообще
Для произвольного конечного в ерене точно есть.
>> No.162669  
>>162664
> > что эта задача решена даже для плоскости
> В смысле, что площадь не у любой фигуры определена? Ну да, не у каждой.


В смысле, что задача, о которой ты говоришь, не решается. В смысле, что нет, насколько мне известно, общего способа построения кривой с алгебраической параметризацией, проходящей через данные точки и заметающей минимальную площадь. Если ты знаешь такой способ, зачем спрашиваешь?

> Для произвольного конечного в ерене точно есть.

Тут ты прав, я почему-то подумал не от тех кривых, когда это писал.
>> No.162671  
>>162669
Я мимо, задача того товарища решается условной минимизацией, хотя не совсем понял, что он в этой жизни хочет.
>> No.162673  
410016038773477
>> No.162674  
>>162671
> задача того товарища решается условной минимизацией

Гм. Ты готов написать соответствующее уравнение вариации? Даже если оно будет решаться в квадратурах, решение будет, вообще говоря, только гладким, а нужны полиномы.
>> No.162679  
>>162674
Ну, если речь про 2д, задача всегда может быть решена построением интерполяционного полинома и умножением на какое-то разложение по лежандрам с минимизацией интеграла от модуля полученного выражения на данном интервале по коэффициентам разложения, при этом соответствующие уравнения тривиальны. И это только первое, что приходит в голову.
>> No.162680  
>>162679
Как минимизировать интеграл функции, график которой проходит через данные точки, мы знаем, спасибо. Вот только задача не в этом. Вопрос в том, как минимизировать площадь внутри параметризованной кривой.

Потом, твое решение дает только минимум на многочленах данной степени. А если на многочленах более высокой можно достичь меньшей площади?
>> No.162681  
>>162680
Что значит твое "площадь внутри кривой"? Если ты минимизируешь в 3д площадь внутри контура, это дает вполне определнный диффур, который всегда можно решить. А последняя фраза вообще смысла не имеет, так как многочленами можно приблизить вообще любую непрерывную функцию, и минимизация оптимальной для контура площади по всем таким контурам даст какую-то убогую форму, которая ничему разумному не соответствует.
>> No.162684  
>>162681
> Что значит твое "площадь внутри кривой"?

То и значит. Интеграл от формы объема по фигуре, ограниченной кривой. Можно переформулировать через теорему Стокса, как интеграл по самой кривой от определенной дифференциальной формы.

> Если ты минимизируешь в 3д площадь внутри контура, это дает вполне определнный диффур

> и минимизация оптимальной для контура площади по всем таким контурам даст какую-то убогую форму

Таки я не понимаю. То у тебя задача оптимизации дает "вполне определенный диффур, который всегда можно решить", то она дает "убогую форму, которая не соответствует ничему реальному". Видимо, ты говоришь о двух разных задачах, вот только ты ни одну из них как следует не определил.

> так как многочленами можно приблизить вообще любую непрерывную функцию

Это, кстати, good point, только нам требуется аппроксимация в другой метрике - где в виде расстояния выступает площадь между двумя контурами.
>> No.162685  
>>162684
Захотелось грубо выругаться из-за тебя. Написал же, что для данного замкнутого контура диффур решается, если соответствующая поверхность ориентируема, а для семейства контуров, по которым минимизируется такая минимальная площадь, область сходится к какому-то убогому звездчатому множеству с искомой границей, проходящей через все данные точки. Ты студентота 1-2 курса что ли?
>> No.162687  
>>162685
Какую задачу ты решаешь? Скажи пожалуйста, точно и ясно.

Насколько я понимаю, в исходных постах речь шла о поиске для данного набора точек замкнутой кривой, параметризованной многочленами, проходящей через эти точки и заметающей минимальную площадь. Ты, насколько вижу, говоришь о какой-то другой задаче, где тебе дан некий замкнутый контур, по которому ты что-то минимальной площади ищешь.
>> No.162688  
>>162687
Кажется, уже отписался тебе, что выше не мои посты, и я ничего не решаю. И касательно твоей постановки, дескать минимальная площадь у контура, я так же все написал, добавив, что задача некорректна сформулирована.
>> No.162748  
>>43626 - первый
>>55775 - второй
>>66948 - 3
>>74216 - 4
>>77310 - 5
>>80961 - 6
>>84296 - 7
>>87526 - 8
>>90593 - 9
>>93583 - 10
>>98491 - 11
>>99255 - 12
>>106104 - 13
>>109788 - 14
>>112885 - 15
>>119972 - 16
>>125166 - 17
>>130172 - 18
>>133509 - 19
>>136767 - 20
>>141253 - 21
>>144684 - 22
>>151715 - 23
>>157896 - 24
>>160159 - 25
>> No.162773  
Анон, объясни мне введение параметра в диффернциальных уравнениях. Лучшее вообще про этот метод, но я могу пока сформулировать только про диффуры 1-го порядка. Почему мы просто берем и приравниваем производную к какому-то параметру? Как это работает? Почему бы тогда не представить x как параметр? Или у?
>> No.162796  
science-vs-math.png (0.0 KB, -1x-1)
0
Вот я спрашиваю в гугле, как связаны пи и е — и мне рассказывают, что:

Константа е непосредственно связана с однородностью пространства и времени, а - с изотропностью пространства. Тем самым они отражают законы сохранения: число е - энергии и импульса (количества движения), а число - вращательного момента (момента импульса). Обычно столь неожиданные утверждения вызывают удивление, хотя по существу, с точки зрения теоретической физики, в них нет ничего нового. Глубинный смысл этих мировых констант остается terra incognita для школьников, студентов и, по-видимому, даже для большинства преподавателей математики и общей физики, не говоря уже о других областях естествознания.

https://www.nkj.ru/archive/articles/4774/
https://www.ilyabirman.ru/meanwhile/all/pi-and-e/
https://www.written.ru/articles/science/complex_exponent
https://www.aif.ru/society/education/chtoosobennogovchislepiotvechaetmatematik

Обсуждаем.
>> No.162797  
>>162796
> геометрические свойства пространства связаны с геометрическими свойствами пространства
> Обсуждаем.

Ну ок.
>> No.162799  
>>162797
Разъясните же троечнику. Это доказательство родства русских с этрусскими, да?
Или теорема Чебышёва о распределении простых чисел, написанная понятным языком.
>> No.162802  
>>162799
> Или теорема Чебышёва о распределении простых чисел, написанная понятным языком.

Где ты ее там увидел?
>> No.162809  
>>162796
Глупо связывать математические константы с физическим миром. В этом мире их нет, они есть только в физических моделях, которые были также выдуманы, как и сами они.
>> No.162987  
Допустим у нас имеется функция F из R^2 в R. Известно, что при фиксации любой из переменных функция является многочленом. Является ли функция F многочленом от двух переменных ?
(спасибо всем кто поможет)
>> No.162992  
>>162987
https://math.stackexchange.com/questions/606523/real-polynomial-in-two-variables
Спасибо, сам нашёл.
>> No.162995  
m3KDUaZgze0.jpg (0.0 KB, -1x-1)
0
Финалисты конкурса "Улыбка аспергера - 2018".
>> No.163010  
>>162995
Кто справа? Вербицкий начинает на Гусейна-Заде походить с возрастом.
>> No.163012  
>>162748
Вспомнил молодость -- перечитал эпичный тред, где бородатый фрик учил определению градиента.
>> No.163013  
>>163012
Где-то он теперь, этот клёвый фрик-сан.
>> No.163112  
Посоветуйте какую-нибудь годную книгу по цифровой обработке сигналов.
>> No.163118  
frSyvVwg2gI.jpg (0.0 KB, -1x-1)
0
Где почитать про связь поиска собственных векторов и решение диффур? Или если вы можете, то объясните сами.
>> No.163119  
>>163118
В.И. Арнольд, "Обыкновенные дифференциальные уравнения".
>> No.163325  
Можно ли построить функцию R^2 -> R такую что она бесконечно дифференцируема вне нуля, в нуле её первые и вторые чистые частные производные непрерывны, а вторая смешанная(хотя бы одна) неограничена.
Спасибо.
>> No.163327  
>>163118
Диффуры мего хуево идут, если ты сдал интегралы на три, потому что суть дифуров в ядренном интегрировании. Чтобы сдать диффуры, снова читни про интегралы лучше.
>> No.163408  
1514915164674.jpg (0.0 KB, -1x-1)
0
>>163327
Я понимаю, конечно, что отвечаю на немного прокисший пост, просто только сейчас увидел, но я не сдаю диффуры. И я сдал матанализ на отл в свое время. И диффуры тоже сдал на отл. Никаких интегралов я не сдавал, сомневаюсь также, что суть диффур в "ядренном интегрировании". Мне просто интересно было почитать с точки зрения геометрии о таких задачах. А также какую-нибудь инфу о таких приколах, например, как в квантовой механике, задача на решение уравнения Шреденгера + поиск собственных функций операторов.
Спасибо тому анону за Арнольда бтв.
>> No.163410  
>>163408
Кое-что есть в книге «Linear Algebra and Its Applications» (David C. Lay), вот только я половины не понимаю там, я похоже гуманитарий.
>> No.163532  
Анон, подскажи, "метод анализа гомотопий" (https://en.wikipedia.org/wiki/Homotopyanalysismethod) это годная тема, которая затыкает за пазуху всю возможную теорию возмущений или мутная хуйня от мутных китайцев?
>> No.163534  
>>163532
Автор мутный. Пишут ноунеймы какие-то.
http://numericaltank.sjtu.edu.cn
>> No.163538  
>>163532
Я ни разу не спец в матфизике и диффурах, но статья очень мутная. Большую часть объема занимают похвалы этому волшебному методу.

> The HAM is an analytic approximation method designed for the computer era with the goal of "computing with functions instead of numbers."

> In the last twenty years, the HAM has been applied to solve a growing number of nonlinear ordinary/partial differential equations in science, finance, and engineering

> the HAM gives excellent flexibility in the expression of the solution and how the solution is explicitly obtained

Впервые вижу математическую статью в википедии, написанную таким языком. Видимо, китайцы и писали.
>> No.163611  
--454.jpg (0.0 KB, -1x-1)
0
>>160159
Уважаемые собеседники!
Я хочу понять предпосылки и суть вопросов, подвигнувших на математические исследования, а также и вопросы, возникавшие в течении этих исследований и направлявшие их.
Например, из-за ОКР и характерной обеспокоенности "правильностью" мне понятно стремление к формализаторству. Еще я понимаю натуральные числа как представление о бесконечности, а точнее счетной бесконечности – впрочем, некоторые философские течения и не признают иной.
Однако даже из немногих известных мне математических понятий не все находят в моем сознании опору в виде понимания их подоплеки, поэтому такие понятия оказываются для меня существующими лишь отдельно сами по себе.
Наверное, книги вроде трудов Бурбаки охватывают широкую область, но не направлены на помощь в моей ситуации. А какие же направлены? Или как мне разобраться в логике исследований и происхождении понятий?
>> No.163614  
>>163611
> Однако даже из немногих известных мне математических понятий не все находят в моем сознании опору в виде понимания их подоплеки
Пример. И можешь выражаться не так глупо и пафосно?
>> No.163618  
>>163611
Одно типичное направление исследований - исследование адекватности формальных определений умозрительным концепциям. Например, у нас есть представление о линии (это что-то типа ниточки) и о поверхности (это что-то типа листка бумаги). Как определить линию и поверхность? Вопрос нетривиален. Ибо можно, например, взять квадрат, поделить его на девять клеточек (как при игре в крестики-нолики) и выбросить центральную клеточку, а потом рекурсивно проделать эту же процедуру с каждой из восьми оставшихся клеток до бесконечности. В результате получится нечто, что невозможно априори считать поверхностью; но и линией этот объект сходу признать нельзя. Определения должны отвечать, к какому из двух классов относится этот объект, и притом отвечать убедительно, резонно. Поиск определений и изыскание патологических объектов очень интересен.

Другое типичное направление исследований - классификация каких-то объектов с точностью до изоморфизма. Например, можно спросить, сколько качественно разных векторных пространств размерности n есть над полем вещественных чисел? Ответ удивляет своей простотой: одно и только одно. То есть по существу имеется лишь одно вещественное векторное пространство размерности 87, например. Или вот другой вопрос. Сколько принципиально разных эндоморфизмов есть у данного комплексного векторного пространства? Ответ на этот вопрос менее прост и даётся теорией так называемой жордановой формы матрицы.
>> No.163621  
Анон, а какое пространство будет двойстенным к пространству полилинейных форм?
А к пространству многочленов степени n?
>> No.163622  
>>163614
Ну вот, например, группа. Или векторное пространство.
Но я не хочу сейчас увлекаться рассмотрением в треде этих частностей. Мне нужен способ прояснения широкого круга понятий.
Например, совет читать такие-то книжки, что вот в такой-то книге объясняют понятие группы, а в такой-то – линейная алгебра.
Вполне допускаю, что могу выражаться еще и не так глупо и пафосно.
>> No.163623  
>>163622
Адольф Юшкевич, книги по истории математики.
>> No.163624  
Анон, вопрос по стат. исследованию от полного чайника. Есть три детали A, B и C одного комплекта. Берётся N попыток на получение случайной детали. Максимальное количество полных комплектов, которые можно собрать = N/3. Минимальное = 0, ну не повезло вытащить ни одной детали какого-то вида. Но чувствую я, что и то и то имеет меньшую вероятность, чем что-то посередине. Что можно сказать о вероятности собрать полный комплект?
>> No.163626  
>>163622
> совет читать такие-то книжки, что вот в такой-то книге объясняют понятие группы, а в такой-то – линейная алгебра.
> Или как мне разобраться в логике исследований и происхождении понятий?

Я тоже не совсем понял столь усложнённый канцеляризмами язык, но думаю, тебе пока хочется чего-то вроде философии, а не непосредственного ботанизма.

Попробуй научно-популярную литературу по математике. Тебе должны понравиться:
"Математика. Утрата определённости", М. Клайн
"Математическое мышление", Г. Вейль
"Математика как метафора", Ю. Манин
"Апология математика", Г. Харди
"Плач математика", П. Локхарт
"Прелюдия к математике", У. У. Сойер
"О науке", А. Пуанкаре

Больше практики, меньше философствования:
"Математика и правдоподобные рассуждения", Д. Пойа (что за дурацкая транслитерация)
"Математическое открытие", Д. Пойа (тоже к вопросу о логике исследований)
"Что такое математика?", Р. Курант, Г. Роббинс (часто упоминают тут)

Некоторые алгебраисты советуют эту книжку для нубов, но я не читал:
"Основные понятия алгебры", И. Шафаревич

Историю тебе уже советовали, и на вполне здравых основаниях. Обязательно почитай, чтобы примерно ориентироваться в хронологии и основных проблемах.
Если лень накатывать большие энциклопедии, есть вариант попроще:
"Краткий очерк истории математики", Д. Я. Стройк

> вот в такой-то книге объясняют понятие группы, а в такой-то – линейная алгебра.
Это уже учебники, для усвоения которых нужна серьёзная практика.

Есть немалое количество гайдов во всём интернете, в том числе и на Доброчане, где тебя завалят рекомендациями с книжками. Читай до отвала. Например,
https://sites.google.com/site/scienceandmathguide/subjects/mathematics
>> No.163627  
>>163624
Очевидно, что ~N/3 комплектов ты и будешь получать, если вероятности получить a, b, c равны 1/3.
>> No.163628  
>>163622
Ну вот. Я думал, что речь пойдет о каких-то комплексных числах, но тут все еще проще. И не нужно было писать такой длинный исходный пост, который нам приходится читать.
Что именно в определении группы неясно? Есть определение, просто как две копейки, аналогии можно провести пока и понять что это и к чему это относится. История возникновения конкретного понятия длинна и безинтересна, не думаю, что стоит повторять путь его изобретателей, да и не факт, что у тебя получится это сделать, потому что ты уже знаешь сильно больше чем они. И так со всей историей математики, интересно почитать в каком году кто и что открыл, поразиться размаху ума и пойти дальше.
Более конкретно. Есть посредственные книги, такие что ты там что-то читаешь и не обращаешь внимания на какие-то детали, которые очень существены, а в книге на них пояснений или указаний, и тебе приходится несколько дней решать задачи по теме, сравнивать ответы, потом идти к умным людям и они тебе поясняют, что вот тут в определении очень важное слово, ну или ты до этого допираешь. Такие книги я считаю не очень хорошие, проще было бы потратить абзац текста и описать тонкость определеения и что ломается если оно другое. Так вот, ты подобное встретишь не скоро. Определение группы и всего остального не допускают ничего подобного. Так что возможно стоит задуматься тем ли ты занимаешься? Цель вообще какая?
>> No.163629  
>>163628
> а в книге на них нет пояснений или указаний
Фикс.
Да и вообще че-то я пост накатал как даун, тяжело самому читать, извините.
>> No.163639  
--436.png (0.0 KB, -1x-1)
0
>>163618
Пожалуй, это проясняет пусть не методы, но мотивацию отдельных исследований.
>>163626
> Я тоже не совсем понял столь усложнённый канцеляризмами язык, но думаю, тебе пока хочется чего-то вроде философии, а не непосредственного ботанизма.
Согласись, фраза
> тебе пока хочется чего-то вроде философии, а не непосредственного ботанизма
звучит расплывчато, да и не можешь ты требовать подобной терминологии! Хотя слово "философия" в некотором аспекте подходит, но я писал о предпосылках, подоплеке, сути – а это все же звучит яснее.
> Это уже учебники, для усвоения которых нужна серьёзная практика.
Ведь учебники не объясняют ни предпосылок, ни сути, ни подоплеки. Учебники только учат получать требуемое решение типовых задач.
Скачал все названые тобой книги, преимущественно отыскивая английскую версию. Сайт просмотрел, но указанные материалы не скачивал.
>>163628
> Так что возможно стоит задуматься тем ли ты занимаешься? Цель вообще какая?
Думаю, тебе стоило искать ответ на этот вопрос в моем сообщении прежде, чем
> писать такой длинный исходный пост, который нам приходится читать
. Об определениях я и слова не говорил.
Постараюсь объяснить с помощью примера.
Если стоит задача разделять что-то сплошное и крепкое, то это можно делать, разбивая сильными ударами. Для того, чтобы делить, нужна острая штука. Чтобы удар был сильным, штуку можно взять тяжелую, и еще можно взять длиную ручку и наносить удар вращающим движением, тогда штука будет ударяться с быстротой. Тогда в штуке можно иметь дырку, чтобы туда продевать ручку.
Таковы предпосылки и подоплека, исходя из которых можно придумать топор.
В то же время можно дать различные определения топора. Можно определить топор как изделие, изготовливаемое по известному чертежу из известного материала и с помощью известной технологии. Изготовитель тогда может не знать подоплеки, зачем нужен топор. Само определение может быть формализованым, чтобы изготовить топор мог специальный автоматический прибор.
В этом состоит различие. Меня интересует подоплека исследования, в том числе предпосылки тех или иных понятий и их определений, но не только лишь фрагменты исследования в виде текста определений.
>> No.163640  
>>163639
> В этом состоит различие. Меня интересует подоплека исследования, в том числе предпосылки тех или иных понятий и их определений, но не только лишь фрагменты исследования в виде текста определений.

Для того, чтобы понять подоплеку исследований, нужно сначала понять их суть, тебе не кажется? Ну, положим, скажу я, что подоплекой исследований Дворка-Гротендика-Делиня дзета-функций Хассе-Вейля алгебраических многообразий(одно из крупнейших математических достижений ХХ века) было желание распространить доказательство Вейля с алгебраических кривых на многообразия большей размерности. В свою очередь, доказательство Вейля было интересно тем, что небезызвестная дзета-функция Римана является вариантом дзета-функции Хассе-Вейля. Проблема лишь в том, что объект, дзета-функцией которого является дзета-функция Римана, не есть алгебраическое многообразие над конечным полем, а потому методы Вейля-Дворка-Гротендика-Делиня не очень применимы к функции Римана. Однако, попытки распространить эти методы до ф. Римана не прекращаются, в частности, связанные с "полем из одного элемента" ( en.m.wikipedia.org/wiki/Field with one_element ).

Много ли тебе даст такой текст? А более подробных ты нигде не найдешь, ибо никому из математиков не интересно разжевывать предпосылки своих исследований профанам. Однако, если понять содержание исследований, а также сличить с более ранними исследованиями, то можно составить собственное представление об истории и философии вопроса. И, боюсь, это единственный путь.
>> No.163641  
>>163639
Тогда тебе нужно читать именно книги по истории математики. ПрАсолов недавно очень хороший трехтомник написал, например.
https://sites.google.com/site/prasolovskacatmoiknigi/neizdannoe-v-processe-napisania
>> No.163642  
>>163639
> Постараюсь объяснить с помощью примера.
Не стоит сравнивать математику с рубкой дров. К тому же имея чертеж топора можно вполне себе понять, как эта штука появилась, что ты, кстати, и сделал. Вопрос зачем ввели определение группы в общем-то задать можно. Но его ответ состоит в самом определении группы - это один из самых эллементарных объектов. Его придумали чтобы получить какой-то уровень абстракции.
Ну и самая главная проблема сравнения с топоромю У тебя стоит задача разрубить что-то и думаешь как бы это сделать. В случае с математикой, обычно, ты прешь вперед, открывая новые закономерности, необычные, красивые вещи, вводишь новые определения и смотришь, что на них получается. А потом через несколько десятилетий какие-нибудь физики обнаруживают, что это говно можно применить куда-нибудь. По крайней мере мне видется именно так и я занимаюсь именно такой математикой. Тыкаюсь, решаю, иногда задумываюсь куда это можно применить.
Так цель у тебя какая?
>> No.163643  
>>163639
> подоплека исследования, в том числе предпосылки тех или иных понятий и их определений
История математики внутричерепно. В частности про такую штуку, как комплексные числа, ибо пример простой, и вне контекста "подоплек" и "предпосылок" вообще нихрена не понятно, откуда они взялись и для чего нужны.
> звучит расплывчато, да и не можешь ты требовать подобной терминологии!
Разница между "предпосылками" и "подоплёкой" в твоём сообщении выглядит куда более расплывчатой и запутанной.
У меня складывается впечатление, будто ты хочешь пощупать какие-нибудь "вещи в себе" и увязнуть в дебрях спекуляций, а не ознакомиться с историей и практикой настоящей математики.
> Ведь учебники не объясняют ни предпосылок, ни сути, ни подоплеки.
Верно, но не все. Поэтому интересно читать популярную и историческую литературу о математике. Ознакомиться с мнениями самих математиков. Заняться математикой, ради всего святого. Она изящная, интригующая и вообще не такая, какой её малюют в средних школах и прочих донных учебных заведениях.
>> No.163650  
--630.jpg (0.0 KB, -1x-1)
0
>>163640
> Для того, чтобы понять подоплеку исследований, нужно сначала понять их суть, тебе не кажется?

Твой вопрос подталкивает отметить лежащий в основе разговор подход.
Исследование, математическое понятие – это индивидуальное достижение конкретного автора и является искуственно созданным им на основе его личного мироощущения.
Если стоит цель изучить психологию, конструкты лица, являющегося автором, то можно использовать введенные им математические понятия как источник для такого изучения.
Но если стоит цель формирования собственных математических понятий, то либо нужно иметь собственную мотивацию введения именно таких понятий, либо установить контакт с понятиями автора через общие места.
Иначе математическая теория представляет собой сугубо приспособление, манипулируемое каким-либо известным интерфейсом, навроде программистской библиотеки. Разумеется, это может быть полезно в практической деятельности. Или даже кому-то странному будет занимательным просто манипулировать интерфейсом, не зная содержания. Но говоря о математическом исследовании, я имею в виду осознание содержания и следующее из него воплощение в теории. При этом, подчеркиваю, смысл и форма произвольны как искуственные порождения личного мироощущения автора.

>>163640
> Однако, если понять содержание исследований, а также сличить с более ранними исследованиями, то можно составить собственное представление об истории и философии вопроса.
Сложный обратный инжиниринг в отчаянной надежде на совпадение с понятиями каждого из авторов исследований – другого человека из другой жизни, другого класса и другой исторической эпохи.
> никому из математиков не интересно разжевывать предпосылки своих исследований профанам
Но вот >>163643 говорит:
> Поэтому интересно читать популярную и историческую литературу о математике. Ознакомиться с мнениями самих математиков.
Масса математических статей и книг как раз ставит целью простое разъяснение, поэтому отношу твое высказывание на увлеченность спором, хоть я и не за спором сюда пришел.
>> No.163651  
--448.png (0.0 KB, -1x-1)
0
>>163641
Благодарен, взял на заметку.
>> No.163653  
--634.jpg (0.0 KB, -1x-1)
0
>>163642
> Не стоит сравнивать математику с рубкой дров.
Разве нужно требовать от собеседника учета заведомо неизвестных предпочтений к рубке дров или же их не рубке вместо благодарности за труд облегчить разговор примером?

> имея чертеж топора можно вполне себе понять, как эта штука появилась
Но изготовитель, производящий топор по чертежу, и автоматический прибор, производящий топоры по формализованному заданию, могут и не знать назначение топора и стоящую за идеей его создания подоплеку.
> В случае с математикой, обычно, ты прешь вперед, открывая новые закономерности, необычные, красивые вещи, вводишь новые определения и смотришь, что на них получается. А потом через несколько десятилетий какие-нибудь физики обнаруживают, что это говно можно применить куда-нибудь.
Обнаружение некоего применения физиками через несколько десятилетий не означает, что изначально за математическим понятием ничего не стояло. И заявление о "красоте" математического понятия не отрицает наличия у него происхождения, а только наталкивает на вопрос: в контексте какой проблематики введение понятия было красивым?
> По крайней мере мне видется именно так и я занимаюсь именно такой математикой. Тыкаюсь, решаю, иногда задумываюсь куда это можно применить.
Надеюсь, тебе удается избежать превращения этого занятия в

> просто манипулировать интерфейсом, не зная содержания
>> No.163658  
>>163653
Прости, но мы с тобой не можем найти с тобой общий язык. Ну вот просто не получается и все тут. Все, что могу добавить, что лично я и разу не пользовался библиотеками, основные моменты которых, я не понимаю. Максимум - я могу плохо разбираться в идеях реализации и оптимизации, но это не является главным в этих либах.
>> No.163662  
--657.jpg (0.0 KB, -1x-1)
0
>>163658
Вся суть библиотеки состоит в принципе черной коробки, то есть в том, чтобы знать результат, но не знать реализации.
А суть математики – в изучении реализации, так, чтобы уметь и самому эту реализацию придумать.
Теперь могло стать понятно.
>> No.163666  
>>163662
В математике тоже есть что-то подобное. Например, вместо докапывания до "сути" математических объектов изучаются отношения, возникающие между ними. Или операции над объектами.
>> No.163672  
Анон, есть вопрос по теории вероятностей, я не уверен, что правильно понимаю зависимые события. Для удобства формализую вопрос в виде задачи.
Есть два игровых автомата: Счастливчик и Однорукий Бандит.
Счастливчик приносит выигрыш с вероятностью A. Однорукий Бандит с вероятностью B.
При этом, если игрок заходит в казино и видит, что Счастливчик уже занят, он встает играть за Бандита.
Вероятность того, что счастливчик свободен равна C.
С какой вероятностью игрок придет в казино и выиграет?
В моем понимании рассчитывается это так:
C A + (1-C)* B
Прав ли я?
>> No.163678  
--680.jpg (0.0 KB, -1x-1)
0
>>163666
Значит, предмет разговора – выяснение идеологии, лежащей в основе абстракции.

Приведу примеры.
1) Я затрагивал векторное пространство. Не буду вникать в тонкости, потому что не особо интересовался и не хочу углубляться в "инфраструктуру" какого-либо конкретного определения этого понятия. По этой теме я могу сказать, что векторное пространство касается преобразований, именно линейных преобразований, область которых так сильно была развита. В высшей школе эта тема подчинена изучению курса анализа.
2) Упоминались комплексные числа. Тоже тема за периферией моего интереса. Комплексные числа относятся к разрешимости алгебраических уравнений.
>> No.163681  
wORDN_BOomY.jpg (0.0 KB, -1x-1)
0
Ребят, есть вопрос. Давно волнует, тоже по теории вероятности, анон выше напомнил мне.

Короче, допустим у нас есть дефолтная монетка, на одной стороне "1", на другой "2". Вероятность того, что выпадет "1" = вероятности того, что выдает "2" и обе равны 0.5. Далее допустим я подкидываю ее дважды и мне выпадает оба раза "1". Теперь я снова ее подкидываю, как оценить вероятность того, что выпадет "1"?
С одной стороны, поебать ваще, что там было до этого. Монетка же никаким образом не изменилась? Все еще 0.5 вероятность, почему ей быть другой?
С другой стороны, вероятность того, что монетка трижды выпадет одной стороной равна 0.5 0.5* 0.5 = 0.125.
Допустим событие А - "Монетка выдаст "1" после двух выпадений "1"" и событие В - "Монетка трижды выпадет на "1"" - это два разных события. Соответственно вероятность А - 0.5, а вероятность В - 0.125.
Но схуяли, если с точки зрения эксперимента происходит одно и тоже? Или я чего-то не понимаю?
>> No.163683  
>>163681
Вероятность получить "орёл орёл решка" тоже 0.125. Проблема в том, что ты сначала постулируешь вероятность выпадения орла, а потом эту вероятность пытаешься посчитать с помощью опыта. А так нельзя. Если ты постулируешь, что вероятность орла 1/2,то у тебя есть бесконечно сильное свидетельство, что монетка симметрична. Это свидетельство не перебить никаким конечным числом опытов с монетой. Если ты несомненно знаешь , что монета симметрична, то даже миллиард орлов подряд не поколеблет твою убежденность. Когда ты знаешь, что монетка симметрична, последовательность из миллиарда орлов не больше удивляет тебя, чем любая другая последовательность из миллиарда букв О и Р. Всё такие последовательности равновероятны. Если же ты бросаешь монетку сто раз, видишь сто орлов и делаешь вывод, что вероятность орла близка к единице, - ты не можешь говорить, что у тебя монетка несомненно симметрична. Потому что это противоречит опытам. То есть если ты берешься переоценивать вероятности с помощью опытов, ты не должен говорить про неизменяемые вероятности. См. теорема Байеса, байесианская теория вероятностей, байесианская сеть доверия.
>> No.163684  
1papa.jpg (0.0 KB, -1x-1)
0
>>163683
Да понимаю. Вот про это не подумал:
> Вероятность получить "орёл орёл решка" тоже 0.125.
По тегам почитаю тоже. Спасибо за ответ большое, надо обдумать еще разок, но вроде проясняется.
>> No.163713  
Кто-нибудь шарит в логике? Не могу найти что такое универсальная модель исчисления. Просто полная и корректная? Т.е. та которая верные формулы в верные переводит, а неверные в неверные?
>> No.163714  
>>163713
Универсальным вычислителем обычно зовут машину Тьюринга (и ей эквивалентные).
>> No.163715  
>>163713
Это может относится также и к теории моделей. Если термин исчисление означает математическую теорию, у так понимаемого исчисления могут быть модели. И универсальная – это какая-то из моделей.
>> No.163716  
>>163714
>>163715
> Не могу найти что такое универсальная модель исчисления
> универсальная модель исчисления

Да, я спрашивал про теорию моделей и меня интересует именно определение универсальной можели. Если это важно то в рамках секвенций и моделей на полугруппах с делением.
>> No.163722  
>>163713
https://staff.fnwi.uva.nl/n.bezhanishvili/Papers/Universal-Models.pdf
>> No.163723  
lejbrouwer.png (0.0 KB, -1x-1)
0
>>163611
>>163639
> В этом состоит различие. Меня интересует подоплека исследования, в том числе предпосылки тех или иных понятий и их определений, но не только лишь фрагменты исследования в виде текста определений.
Ну вот пикрелейтед в этом плане не только очень глубоко копал, но и исчерпывающе объяснял, откуда что и зачем в математике, а чему там вообще не место. Его и сейчас опровергнуть нечем. Вот только у него подача материала сложная, его и ближайшие ученики местами не понимали, про современные объяснения его идей и говорить смешно, такую хуйню пишут. Но как по мне, Брауэр это то, что обязательно стоит осилить, если интересна математика.
>> No.163732  
800238736-bashkortostan-ural-mountains-natural-par.jpg (0.0 KB, -1x-1)
0
>>163723
Хорошая мысль. Я как раз разбираю понятие отрицания.
>> No.163733  
in8.jpg (0.0 KB, -1x-1)
0
>>163723
> lejbrouwer.png
Ого, малай! Прочитал первый абзац, и что скажу. Я рассматриваю как раз понятие "обозримого", "ясного", "точного", где как раз важно это twoity, фактически используемое в виде индукции. То есть если две вещи точны, то и обе вместе они представляют точную вещь.
Сейчас это все очень призрачно, но я надеюсь, что мне удастся продвинуться в будущем.
>> No.163737  
1527968821900.png (0.0 KB, -1x-1)
0
>>163723
>>163733
Вот так интуиционизм и распространяется.
>> No.163738  
brouwer.png (0.0 KB, -1x-1)
0
>>163737
> Вот так интуиционизм и распространяется.
Что не так с интуиционизмом?
>> No.163739  
>>163738
Интуиционистами называют себя несколько научных фриков, которые борются с математикой (как умеют). То вещественные числа обличают, то не верят в группы.
>> No.163741  
>>163739
Очень двачерский у тебя ответ. Повышенно поверхностный и претенциозный.
>> No.163742  
Пол_Джозеф_Коэн.jpg (0.0 KB, -1x-1)
0
>>163741
Слова других людей, которые ты читаешь, эмоционально окрашиваются в твоём сознании в зависимости в основном от твоего личного жизненного опыта. Если ты видишь слово, которое кажется тебе дерзким, - что-то в твоей жизни случилось такое, что ты стал связывать это слово и ощущение дерзкости.
>> No.163751  
wallhaven-453747.jpg (0.0 KB, -1x-1)
0
В мозгу 15М неронов и у каждого 15к связей с другими нейронами, как посчитать возможное колличество таких мозгов. Т.е. как посчитать количество графов с М вершинами и Н ребрами выходящими из каждой вершины?
>> No.163752  
>>163751
Подействовать перестановками на вершины и посчитать орбиты, ололо.
>> No.163753  
>>163752
Как связаны с перестановками соединения, ребра графа?
>> No.163755  
>>163751
> В мозгу 15М неронов
У хомячка? У человека только неокортекс 100млрд нейронов.
> и у каждого 15к связей с другими нейронами,
От 2 до 10к синаптических связей в среднем. Кроме синаптических есть эфаптические и другие. Нормально все это не посчитать.
>> No.163759  
>>163755
Отлично. А как все таки решить комбинаторную задачу, которую я сформулировал?
>> No.163761  
>>163759
Я бы каждому нейрону сопоставил всевозможные способы соединиться из него с остальными нейронами, т.е. С^k (n-1). У каждого нейрона вот столько вот способов распустить щупальца. Значит, всего получается (С^k (n-1))^n возможных вариантов.
>> No.163763  
>>163761
Это абсолютно неверно. Ведь после того, как ты из первого нейрона выпустил k щупалец, у тех k нейронов, с которыми ты соединился, нужно будет провести не k, а k-1 щупалец, а на следующем шаге появятся некоторые вершины, из которых нужно будет пустить k-2 щупалец, и некоторые - из которых k-1, и т.д.

Правильно решать, я думаю, так: граф с n вершинами, у которого из каждой вершины выходят k ребер, можно получить, взяв n экземпляров "звезды", в которой из центральной вершины выходят k ребер, и склеив их. Склеивание означает, что мы крайнюю вершину каждой звезды отождествляем с центральной вершиной какой-нибудь другой звезды, и при этом к каждой центральной вершине должно приклеиться ровно k. Это означает, что число графов равно числу сюръективных отображений множества мощности k n в множество мощности n, при которых каждая точка имеет ровно k прообразов. Это равно C(k, kn) C(k, kn - k) C(k, kn - 2k) ... C(k, 2k) * C(k, k), где под C(a,b) я подразумеваю соответствующий биномиальные коэффициент.
>> No.163764  
>>163761
Поломал всю разметку, С^k(n-1) означает число выбора k нейронов из n-1 варианта, или "цэ из эн по ка"
>> No.163765  
>>163759
Понятно, что таких графов может вообще не существовать. Например, граф на трёх точках, в котором из каждой вершины выходит в точности одно ребро, не существует.
>> No.163769  
--166.jpg (0.0 KB, -1x-1)
0
>>163742
>> No.163773  
06-12-14-foto-fotostranik800x831-32.jpg (0.0 KB, -1x-1)
0
>>163763
Ай красавчик. Реально ведь решение. Спасибо большое.
Только добавлю, что ты пояснил, что за С(n,k) и не пояснил, что за граф-звезда.
>> No.163775  
>>163763
По моему, будут посчитаны варинты склеить нейрон самим с собой.
>> No.163776  
>>163763
Погоди. Но так же получается сильно больше вариантов.
Отображения же не произвольные. Надо еще, чтобы если ребро одной из звезд склеивается с центром другой звезды, т это должно и в обратную сторону произойти.
>> No.163777  
>>163775
Ну да, и это тоже. Я теперь сам не очень уверен в своем решении. Надо еще подумать.
>> No.163778  
>>163776
Да, я уж сам заметил эту ошибку. Не секу я в графах.
>> No.163780  
>>163763
Я рассматривал "щупальца" от нейронов как однонаправленные, а поэтому особо друг другу не мешающие. Насколько я знаю, типичный нейрон принимает сигнал от кучи других, а сам посылает только в один. Это безумное упрощение зоопарка нейронов, но тем не менее


> 163759-кун
>> No.163820  
2e4415d0e3bf4046bf.jpg (0.0 KB, -1x-1)
0
Существует ли такая функция/последовательность/ряд/функциональная последовательность/функциональный ряд над множеством комплексных чисел, который сходится, но не к комплексному числу?
>> No.163821  
>>163820
Так как пространство комплексных чисел полно, нет. Вещественные числа также являются комплексными.
>> No.163822  
>>163820
Комплексные числа - полное метрическое пространство, поэтому любая сходящаяся последовательность комплексных сходится к комплексному (мнимая часть может быть нулевой, конечно).

Сходящаяся функциональная последовательность по определению сходится к функции. Функции, вообще говоря, не являются комплексными числами.
>> No.163823  
>>163821
>>163822
Точно, что-то я затупил.
>> No.163826  
>>163822
> Функции, вообще говоря, не являются комплексными числами.
Ну если "вообще говоря", то конечно лул.
>> No.163828  
>>163826
А что, есть иное мнение?
>> No.163829  
>>163828
Не, я к тому что "вообще говоря" подразумевает, что есть исключения. А функции не могут быть числами никогда.
>> No.163830  
>>163829
> А функции не могут быть числами никогда
Ну почему же, могут. Функция — бинарное отношение xRy, сопоставляющее каждому x из области определения единственный y. Множество пар можно рассматривать как бинарное отношение. Одну пару тоже можно рассматривать как бинарное отношение, причём это отношение, очевидно, является функцией.
Комплексное число z = a + ib определяется парой вещественных чисел, а следовательно, является функцией (z(a) = b).
>> No.163831  
>>163830
> Одну пару тоже можно рассматривать как бинарное отношение, причём это отношение, очевидно, является функцией.
А числом?
>> No.163832  
>>163831
Если это пара вещественных чисел, то эту пару можно рассматривать как комплексное число.
>> No.163833  
>>163832
Формулировка "можно рассматривать как число" - это не тоже самое, что формулировка "также является числом".
>> No.163834  
>>163833
Математика это нетипизированный язык, и поэтому в ней «является» часто означает «можно рассматривать». Например, если точка это n-ка чисел, то декартово произведение двух множеств точек не даёт множество точек (потому, что (a (b, c)) ≠ (a, b, c)). Однако, это никого не останавливало от использования теоретико-множественного языка.
>> No.163835  
>>163834
Ладно няша, убедил.
>> No.163837  
>>163829
На некотором множестве функций можно ввести структуру непрерывно упорядоченного поля, например. Поскольку существует, с точностью до изоморфизма, лишь одно такое поле, - эти функции обязательно будут являться вещественными числами.
>> No.163840  
2018-06-15-001033_1366x768_scrot.png (0.0 KB, -1x-1)
0
Нужно нарисовать обычную параболу, вертикальную, без всяких поворотов, но библиотека этого не умеет, но зато умеет в кривые Безье. Проблема в том, что я о них знаю чуть менее, чем ничего. В связи с этим у меня встала пара вопросов, а именно:
Всякая ли кривая Безье второго порядка — кусок параболы?
Есть ли какие-то условия на точки, или они могут быть абсолютно любыми?
Как получить из этого график y = ax^2 + bx + c ? В смысле, P_i — это же точки, как из этого получить привычное уравнение от х? Просто раскрыть скобки, сложить и взять из полученного только одну компоненту?
А в обратную сторону как? Выразить P i через а, b, c, а потом в каждую компоненту каждого P i получается нужно подставить одно и то же? Т.е. получатся точки вида (t, t)?
>> No.163847  
>>163840
Возможно, это тебе поможет:
https://pomax.github.io/bezierinfo/
https://www.jasondavies.com/animated-bezier/
>> No.163852  
>>163847
Спасибо, первый сайт помог. Алсо, как оказалось часть ответов есть на википедии.
>> No.163875  
80021857-7CC0-4994-BECB-23DAB6357C9C.jpg (0.0 KB, -1x-1)
0
Существует такое множество (метрическое пространство?), на котором существует фундаментальная расходящаяся последовательность или сходящаяся нефундаментальная последовательность?
>> No.163876  
>>163875
> фундаментальная расходящаяся последовательность
Расходящихся фундаментальных не бывает, только не сходящиеся.
Фундаментальная последовательность всегда ограничена, даже если она не сходится.
Пруф:
По определению фундаментальной, ∀e>0 ∃N = N(e) ∀m>N ∀n>N r(xm, xn)<e
Положим e=1. Пусть N = N(e), n = N+1. Рассмотрим шар с центром в xn радиуса 1.
Тогда все точки последовательности с номерами, большими N, лежат в этом шаре.
Пусть R - наибольшее из чисел {1, r(xn, x1), r(xn, x2), ... r(xn, xN) }.
Тогда шар с центром в xn радиуса R содержит все точки последовательности.

> сходящаяся нефундаментальная последовательность
Если последовательность сходится, то она фундаментальна.
Обратное неверно.
>> No.163877  
>>163876
Я думал расходящаяся == не сходящаяся.
А есть пример пространства, где есть не сходящиеся фундаментальные последовательности?
>> No.163878  
>>163877
> А есть пример пространства, где есть не сходящиеся фундаментальные последовательности?

Рациональные числа - канонический пример. Последовательность десятичных приближений какого-нибудь иррационального числа фундаментальна и не сходится.
>> No.163879  
>>163877
Q как подпространство R.
Последовательность рациональных чисел, которая в R сходится к корню из 2, фундаментальна в Q, но предела в нем не имеет.
>> No.163880  
>>163879
То, что ее предел не принадлежит Q просто значит, что Q — не полное. Но последовательность­-то все равно сходится.
>> No.163881  
>>163880
В Q - не сходится.
>> No.163882  
>>163881
Т.е. существует е > 0, что для любого N > 0 существует n > N для которого |sqrt2 - x_n| > e?
Но ведь это не так, т.к. любое вещественное число можно сколь угодно сильно приблизить рациональным числом.
>> No.163883  
>>163882
Корень из 2 не является элементом Q. Это иррациональное число.
>> No.163884  
>>163883
Я знаю, и что? В чем проблема-то?
>> No.163885  
>>163884
В R у последовательности есть предел.
В Q у последовательности нет предела.
Последовательность не сходится в Q.
Ты не троллишь?
>> No.163886  
>>163885
Все, теперь загуглировал и понял. Оказывается, наличие предельной точки не означает сходимость.
>> No.163887  
>>163886
Нет, ты не понял. В рациональных числах у этой последовательности и предельных точек нет.

Если последовательность рациональных чимел сходится к иррациональному, то как последовательность рациональных чисел она не сходится. Предел должен принадлежать тому же топологическому пространству, что и элементы последовательности(иначе можно было бы утверждать, что последовательность 1, 2, 3, 4, 5, .... сходится, т.к. к действительным числам можно добавить элемент "бесконечность").
>> No.163894  
В чем отличие стягиваемости по и просто стягиваемости?
Изучаю все того же фоменко фукса.
Стягиваемость - гомотопическая эквивалентность точке
Стягиваемость X принадлежащее Y (не понятно множество или пространство x, в определении про пространство, а дальше про множество, тоже поясните) это эквивалентность вложения X в Y(видимо тождественное отображение, в смысле мы рассматриваем X как отдельное множество и его элементы отображаем в элемениы X как подмножества Y тождественно) и отобажения X->Y, которое переводит X в точку.
Категория люстерника-шнирельмана, это наименьщая мощность покрытия X замкнутыми множествами(вот про это я и говорил, в определении для пространств, а тут для мнлжеств), такими что они стягиваемы по X.
Если в конце стягиваемость по заменить на обычную, то получится сильная категория.
Почему первая категория является гомотопическим инвариантом, а вторая нет?
>> No.163899  
>>163894
> не понятно множество или пространство x

Любое подмножество топологического пространства - тоже топологическое пространство, на нем индуцированная топология . Вообще в топологии множества без топологической структуры практически не рассматриваются(только как вспомогательные), поэтому "множество" и "пространство" можно считать синонимами.

Разницу между стягиваемостью и стягиваемостью по множеству можно проиллюстрировать так: существует вложение прямой в тор(т.н. иррациональная обмотка тора), при которой прямая в торе всюду плотна, т.е. ее замыкание совпадает с самим тором. При таком вложении прямая по тору не стягиваема: поскольку прямая в торе всюду плотна, любоое отображение прямой в тор непрерывно продолжается до вложения тора в тор, и тогда стягиваем был бы сам тор.
>> No.163905  
>>163899
Почему тор не стягиваем я пока не знаю, вообще не знаю примеров несиягиваемых пространств, но идею уловил. Выходит стягиваемость по более строгое условие, вроде как. Впрочем да,это ж очевидно.
Спасибо большое, внес порялок, сейчас попытаюсь разобраться с категориями.
>> No.163906  
>>163887
Но как же проколотая окрестность? Множества, не являющиеся замкнутыми, т.е. содержащие не все свои предельные точки?
>> No.163907  
>>163906
Этот вопрос ты задаешь по какому поводу?

В проколотой окрестности не будет сходится последовательность, имеющая пределом выколотую точку. Ты, по-моему, не понимаешь одной вещи: сходимость - свойство не только самой последовательности, но и топологического пространства, как часть которого мы эту последовательность рассматриваем. Поэтому нет ничего особенного в том, что одна и та же последовательность сходится в R, но не сходится в Q.
>> No.163914  
Я-тупой.png (0.0 KB, -1x-1)
0
Решите простенькую задачку для гуманитария! Пожалуйста!
Да, я тупой!
>> No.163915  
>>163914
Введи в калькулятор arcsin(76/290)
>> No.163947  
Приветик.
Решил вот летом вкатиться в математику, а то уже второй курс, а понятия никакого. Кратные ряды, интегралы, диф. уравнения и прочее для меня совсем темный лес. Посоветуйте книг для самоизучения. Я не тупой, но отсутствие базы, которую я проебал в школе, очне сказалось на том, что имею сегодня.
Конкретно планирую начать с 10 класса.
На счет книг: на одном сайте двощ советуют книги 70х годов, но конкретных авторов не указывают.
Может вы подскажите фамилии тех людей, чьи книги будут полезны?
>> No.163950  
>>163947
> Конкретно планирую начать с 10 класса.

Начинать с десятого класса тебе совершенно не нужно. Школьная математика очень слабо связана с математикой высшей. Если ты хочешь изучить именно

> Кратные ряды, интегралы, диф. уравнения и прочее

то изучение школьного материала десятого класса в этом тебе не поможет никак, только время потеряешь. Книги могу посоветовать такие, для начала:

Зорич, математический анализ.

Гельфанд, линейная алгебра.

Арнольд, обыкновенные дифференциальные уравнения.

> Я не тупой, но отсутствие базы, которую я проебал в школе, очне сказалось на том, что имею сегодня.

Нет никакой базы, я наблюдал множество школоотличников(вплоть до призеров всероссийских олимпиад), которые не смогли перестроиться на изучение настоящей математики.
>> No.163951  
>>163950
Ты сам-то эти книги читал?
>> No.163952  
>>163951
Эти и еще множество других. А что?
>> No.163953  
>>163950
Благодарю за книги.

> не смогли перестроиться на изучение настоящей математики.
Как перестроиться? Вникать в задачки? Использовать абстрактное мышление?

> Школьная математика очень слабо связана с математикой высшей
Однако,там есть логарифмы, которые я совсем пропустил. Да и ощущение такое, блто большая часть ондногрупников еще в школе разбирали интегралы, а на меня они лишь в универе свалились.
>> No.163954  
>>163953
> Однако,там есть логарифмы, которые я совсем пропустил.

В Зориче логарифмические и степенные функции определяются с нуля. Если знаешь тригонометрические, этого вполне достаточно.

> Как перестроиться? Вникать в задачки? Использовать абстрактное мышление?

Твои вопросы крайне расплывчаты. Ты можешь четко определить, что такое "вникать в задачки"? В любом случае, я не берусь судить о внутренних состояниях других людей. Но факт в том, что они не смогли, несмотря на прекрасную школьную базу.
>> No.163955  
>>163953
Я бы с опаской относился к этому совету. Зорич то там может что и определяет, но я подозреваю, что делается это с оглядкой на то, что человек понимает о чем идет речь. Возможно лучше обойтись без чтения учебников, а быстро пролистать сайты со школьной математикой, благо их куча.
Есть еще одна причина сделать это, а точнее продолжение предыдущей. В учебниках все спразу же начнут ловко опререривать с простейшими понятиями, которые ты, может и знаешь, но не работал с ними никогда.
>> No.163956  
>>163950
> Арнольд
> для начала

Я бы сдох на его месте.
>> No.163957  
>>163956
>>163955
Ну так предложите свой вариант, хуле.

> В учебниках все спразу же начнут ловко опререривать с простейшими понятиями, которые ты, может и знаешь, но не работал с ними никогда.

Назовите мне хоть один учебник, хоть по школьной программе, при котором такая проблема не возникает. При листании сайтов со школоматематикой так не происходит только потому, что на этих сайтах нет теории.
>> No.163959  
>>163952
Не верю.
>> No.163960  
>>163957
Особенно мне интересн более простой вариант по диффурам. Арнольда можно обвинять много в чем, но, по-моему, в наглядности и понятности этой книге нет равных.
>> No.163961  
>>163957
Не могу ничего предложить. "Начала" проходил на парах в университете, на лекциях и семинарах, книги только уже после начал читать, когда руку набил.
> При листании сайтов со школоматематикой так не происходит только потому, что на этих сайтах нет теории.
Может в начале теорию тогда отодвигать на второй план?
>>163960
Я диффуры (без теории, чисто методы решения уравнений) проходил в универе по задачнику Филипова. После того, как пощупал все это сам, переходить к Арнольду намного легче.
>> No.163962  
>>163961
> Может в начале теорию тогда отодвигать на второй план?

А чему мы учимся-то? Тупо методам решения задач? Тогда теорию можно не на второй план, а просто в окошко выкинуть. Заучить производные/интегралы/методы решения диффуров, и поехали. Я все же в своих рекомендациях исходил из того, что человек хочет что-то понять. Даже если исходить из того, что непременно нужно "набивать руку" - во всех перечисленных книгах есть упражнения.
>> No.163963  
>>163962
Ничему не учимся. Но перед тем как реально начать учиться, надо немного подготовиться. Тем более во всех книгах, которые ты написал, насколько я знаю, надо владеть хотя бы на минимальном уровне теорией множеств. Сомневаюсь, что этот поц в курсе про то, как это работает, если собрался материал 10 класса повторять. Энивей, книги ты назвал хорошие, но думаю он придет к ним только через некоторое время (если не забьет хуй).
>> No.163981  
Screenshot_62.png (0.0 KB, -1x-1)
0
Приветик. Смотрел решение диф. ур. на ютубе и не понял одно действие. не понял, как он из (1-2х) получил (х-х^2).
Объясните на пальцах, не могу понять.
>> No.163983  
>>163981
интеграл от x^n равен (1/(n+1))x^(n+1) плюс константа
>> No.163985  
>>163983
При чем тут интеграл x^n? Его тут нет.
Тут интеграл из разности 1 и 2x.
>> No.163986  
>>163985
Интеграл от суммы a и b - это интеграл от a плюс интеграл от b.

Интеграл от 1 - это x
Интеграл от -2x - это -x^2
плюс константа
>> No.163988  
>>163986
Все, я сообразил. Извиняюсь.
>> No.164044  
--253.jpg (0.0 KB, -1x-1)
0
>>163953
Энциклопедия элементарной математики, том где про теорию множеств, алгебру и числовые системы.
Или сложнее Феферман. Числовые системы.
Или любые подобные вводные курсы.
>> No.164296  
Безымянный-6.png (0.0 KB, -1x-1)
0
анон как это доказать ?
>> No.164297  
>>164296
По определению равенства и подмножества.

Пусть y принадлежит f(A∪B).
Тогда существует такое x, что x принадлежит A∪B и f(x) = y.
Тогда x принадлежит A или x принадлежит B.
Тогда y принадлежит f(A) или y принадлежит f(B).
Тогда y принадлежит f(A)∪f(B).
Значит, f(A∪B) ⊂ f(A)∪f(B).
>> No.164299  
>>164297
пасиб
>> No.164300  
1497463952608.png (0.0 KB, -1x-1)
0
Читаю ОДУ Арнольда. Чем отличается поле векторов от поля направлений?
>> No.164301  
>>164300
У вектора есть длина и знак, у направления - нет. Три вектора (2,4), (1,2) и (-2,-4)(на плоскости) задают одно и то же направление. Каждому векторному полю соответствует некоторое поле направлений, но не наоборот. В книжке Арнольда есть пример поля направлений, которому не соответствует никакое векторное - рассмотри этот пример повнимательнее.
>> No.164302  
>>164301
Точно, спасибо, пример рассмотрю.
>> No.164303  
>>163950
> Начинать с десятого класса тебе совершенно не нужно. Школьная математика очень слабо связана с математикой высшей.
> Нет никакой базы, я наблюдал множество школоотличников(вплоть до призеров всероссийских олимпиад), которые не смогли перестроиться на изучение настоящей математики.

Как такое возможно? Ты можешь аргументированно обобщить свои наблюдения и написать в развёрнутом посте? Что значит "перестроиться" и почему возникает такая проблема?

Заглядываю иногда по своему гуманитарному любопытству в советские учебники по математике и научпоп серии "квант". Сталкиваюсь с тем, что авторы сходу используют непростую терминологию, не объясняя, почему и для чего она существует, почему именно в таком виде. Забавно, что "пререквизитные" школьные учебники тоже мало чем обосновывают введение новых определений, не позволяя проследить их эволюцию и смысл.
Разумеется, это недоразумение можно исправить тем же научпопом, но осадок после сугубо "технических" уроков математики остался.

Мунин на dxdy говорит, что нужно читать по порядку школьные учебники, ибо они хорошо справляются с задачей подготовки к "настоящей математике".
>> No.164304  
>>164303
> Мунин на dxdy
Некультурно.
>> No.164305  
>>164304
Бросать туманные комментарии с какой-то стороны тоже некультурно.
>> No.164306  
>>164303
> Мунин на dxdy говорит

А кто это такой, и почему его мнение чего-то стоит? По-моему, ты не очень разборчив в источниках информации. В интернете можно найти советы от крупнейших работающих математиков, вплоть до филдсовских лауреатов, а ты ищешь мудрости на сайте, который не выделяется уровнем даже среди русскоязычных.

> Сталкиваюсь с тем, что авторы сходу используют непростую терминологию, не объясняя, почему и для чего она существует, почему именно в таком виде.

Тут в тебе проглядывает гуманитарий. Значение слова есть его употребление - проследи, как и зачем какое-либо понятие используется, и ты поймешь, что оно значит.

> почему и для чего она существует, почему именно в таком виде.

Смыслом существования того или иного математического понятия является теория, которую можно построить, основываясь на этом понятии. Как сказал один умный человек, математика есть искусство манипулирования понятиями, созданными специально для этого манипулирования. Иными словами, для того, чтобы понять, зачем нужно понятие, введенное в начале математический книжки, и почему его именно такое, ты должен прочитать всю книжку целиком, а не ждать, что тебе сейчас в коротенькой сноске изложат смысл понятий, создававшихся столетиями. Да, это долго, но в геометрии нет царского пути.

> Как такое возможно?

Как возможен разрыв между школоматематикой и математикой настоящей?

1)школьная программа КРАЙНЕ архаична. Посмотри школьный учебник по геометрии девятнадцатого века, и ты увидишь, что он по содержанию очень мало отличается от нынешнего. А математика, естественно, все это время двигалась вперед. Все, что есть общего у школоматематики и настоящей - логический каркас, т.е. методы доказательств, критерии строгости рассуждений и т.п. Но как раз эти вещи в школьной программе задвинуты на второй план, куда больше внимания уделяется вычислительным аспектам.

2)собственно, такая ситуация не только в математике. Если бы ты пришел на форум историков и спросил, что бы почитать по истории, то, уверен, тебе тоже мало кто посоветовал читать школьные учебники.
>> No.164307  
>>164306
> Значение слова есть его употребление - проследи, как и зачем какое-либо понятие используется, и ты поймешь, что оно значит.
Справедливости ради, авторам следовало бы больше пользоваться примерами.
> Смыслом существования того или иного математического понятия является теория, которую можно построить, основываясь на этом понятии.
Из теории обычно можно выделить подтеорию, обосновав понятия гораздо раньше конца книжки.
>> No.164308  
>>164307
О каких книгах мы говорим, в конце концов? Пока разговор чрезмерно общий и потому бестолковый. Единственные книги, в которых примеров нет совсем - книги за авторством Бурбаки. Во всех остальных книгах, которые я читал, примеры были в количестве. Ну так Бурбаки обычно никто в качестве учебника и не рекомендует.
>> No.164309  
>>164308
Не читал Бурбаки - не говори про Бурбаки. У них даже картинки есть.
>> No.164310  
>>164309
> У них даже картинки есть.

Дорожные знаки, лол?

Я не говорил, что во всех книгах всех Бурбаки нет примеров, я утверждал только то, что все книги без примеров, которые попадались лично мне - были авторства Бурбаки. Следует также оговориться, что под Бурбаки я имею в виду не только книги, подписанные "генерал Бурбаки", но и книги, написанные участниками группы и подписанные их собственными именами. Особенно отсутствием примеров ставятся книги Артина и Шевалле.
>> No.164311  
Сколько примерно лет уйдёт на прочтение учебника типа "Математический анализ" Зорича?
>> No.164312  
>>164310
Нет, натурально картинки. Например, в первой книге по общей топологии у них есть картинка открытого множества. Вообще, а в какой это книге у них примеров нет?

>>164311
Если мотивирован, за полгода точно прочитаешь.
>> No.164313  
>>164311
Читать его целиком совершенно не обязательно. Но если прямо отвечать на вопрос, то обычно на этот материал уходит года два. Курс Львовского по анализу рассчитан на три семестра, т.е. полтора года, но он сильно ужат, да и автор, как сам признается, писал его для исключительно сильных студентов.
>> No.164314  
>>164312
> Вообще, а в какой это книге у них примеров нет?

Напомню, что под Бурбаками я имею в виду всех людей, которые входили в группу "Бурбаки". Так вот, примеров нет, например, в небезызвестном "курсе арифметики" Серра, да и вообще во многих его книгах. Еще примеры? "локальная алгебра" того же Серра, "теория полей классов" Шевалле, многие книги Артина(который примеров избегал принципиально). Впрочем в "курсе арифметики" таки есть целая одна картинка.

> Если мотивирован, за полгода точно прочитаешь.

Ох лол.
>> No.164315  
p0086.png (0.0 KB, -1x-1)
0
>>164314
Арифметика Серра - это просто подробное рассмотрение отдельных частных штук, таких как символ Лежандра. В менее специализированных курсах они были бы примерами. То есть вся книга является не более чем очень разросшимся списком примеров и частных случаев. Но и специальное слово "примеры" в ней встречается.
> Ох лол.
Судя по этой фразе, ты мотивированным не являешься. Ну не все же такие.
>> No.164317  
>>164315
> Но и специальное слово "примеры" в ней встречается.

Как и картинки, 1 раз, на 90 странице из 180? Надеюсь, ты понимаешь, что это сильно меньше, чем считается нормальным?

Бурбаки приводят мало примеров в своих книгах - это мнение общепринятое. То, что ты смог полнотекстовым поиском найти целых три примера на середине двухсотстраничной книги - как раз то исключение, которое подтверждает правило.

> Судя по этой фразе, ты мотивированным не являешься.

Да и не стремлюсь, я так, мимо проходил. Однако весь Зорич за полгода - это сильно даже по меркам интернета. Даже Вербицкий весь курс анализа в своей программе не пытался ни в один, ни в два семестра уместить. А мы все же изначально давали советы человеку, который к математике не имеет никакого отношения, так что это нереально вдвойне.
>> No.164318  
Почему у вас столько разговоров о Вербицком?
>> No.164319  
>>164317
> смог полнотекстовым поиском
Просто я эту книгу читал. И даже не для того, чтобы выпендриваться на бордах.
> исключение, которое подтверждает правило
Ты используешь этот фразеологизм неправильно.
> весь Зорич за полгода
Вполне реально. Нужно всего лишь читать в четыре раза больше, чем читает обычный, никак не мотивированный студент.
> не пытался ни в один, ни в два семестра уместить
Это утверждение неверно.
http://verbit.ru/Job/HSE/Curriculum/all.txt
Анализ в объёме Зорича - как раз первые два семестра.
>> No.164320  
>>164319
> Анализ в объёме Зорича - как раз первые два семестра.

Давай теперь я буду ловить тебя на не вполне точных формулировках. Практически весь(за исключением тем со звездочкой)блок "Теория меры и основы функционального анализа", начинающиеся в третьем семестре, состоит из материала, присутствующего в Зориче. Либо ты не знаком с Зоричем, либо с программой, на которую ссылаешься.

> Ты используешь этот фразеологизм неправильно.

Я использую ее в соответствии с узусом, т.е. так, чтобы быть понятным.

То, что с точки зрения филологии узус неправильный, а изначальная римская пословица имела противоположное значение, мне известно.
>> No.164321  
>>164319
>>164320
Да и дифференциальные формы у Вербицкого начинаются только в третьем семестре. Подозреваю, что в программе ты читал только названия блоков.
>> No.164325  
>>164303
Тебе >>164044 хороший совет дал. Начни заниматься и и со временем станешь лучше понимать.
>> No.164328  
Анон, подскажи какой- нибудь канал по математике, где подробно разжевывают каждое действие, каждое сокращение и прочее. Где нет:"Ну тут все очевидно, будет..., А тут ясно, значит, что...".
>> No.164330  
>>164328
Я не знаю, что такое канал. Но я добрый и если ты выложишь примеры, то возможно, я помогу тебе.
>> No.164332  
>>164328
Обожаю эти фразы! Особенно когда «очевидна» страница-другая выкладок.
>> No.164333  
>>164330
Канал на ютубе. Надо было уточнить.
Есть вот такой канал - https://www.youtube.com/channel/UCTx46B991tGoUfCey0Emp1A
Тут парень показывает решения задачек, но он не рассказывает о простых действиях и я часто не понимаю (лично моя проблема же), как он перешел от состояния примера А до состояния Б, что вызывает у меня жопоболь и нежелание смотреть дальше.
>> No.164340  
>>164333
Давай конкретные примеры. Пусть будет скриншот или видео с таймкодом. Будем разбираться.
>> No.164362  
Screenshot_240.png (0.0 KB, -1x-1)
0
>>164340
>>164340
Вот такое уравнение https://youtu.be/tNy3B457PnA?t=3m
Я понял, как он из дроби ux/x(1-u) получил (u/1-u)-u, а потом не понимаю вообще.
>> No.164364  
>>164362
Надеюсь, смогу тебе помочь.
Мы имеем дело с такой вещью, которая составлена из операций и объектов.
Операциями являются =, × (его написание пропущено: вместо u'×x написано u'x), /, -, +, ', …², почти все для получения объекта комбинируют два объекта, и только последние две составляют объект из одного объекта. Причем = создает объект типа формула из объектов типа терм, все остальные из объекта типа терм создают объект типа терм. Объектами являются u, u' (получаемое из u с помощью '), x, u'x (получаемое из u' и x с помощью ×) и т.п. Это термы. Тот объект, который получен с помощью = – это формула.
Операции создания термов однозначны, a=b и c=d, то a+c=b+d. Есть еще другие правила: ассоциативность, дистрибутивность. Отсюда и исходят при преобразованиях.
>> No.164367  
>>164364
Благодарю, анон. Честно, ничего не понял.
Но тут было простое сложение дробей. -u умножили на знаменатель первой дроби (1-u) и в итоге получилось -(u-u^2).
>> No.164396  
Как строго доказать законы де Моргана? Непонятно откуда отталкиваться, какие шаги разрешены аксиомами, а какие нет.
>> No.164402  
>>164396
Доказать – это сделать понятным, поэтому зависит от фоновых знаний. То же самое, формальный вывод происходит в предзаданной формальной системе.
Могу посоветовать посмотреть Huskell B. Curry, Foundations of Mathematical Logic; эта книга переведена на русский язык Хаскелл Б. Карри, Основания математической логики, но мне известен перевод его первого издания, там много ошибок.
>> No.164420  
Patty-ten-ice-creams-please.jpg (0.0 KB, -1x-1)
0
Создавайте новый тред, лентяи. В нём я спрошу: в каком порядке изучать высшую математику?
>> No.164421  
Оцените решение, плиз.

Пусть \{a i\} и \{b* i\} -- две неэквивалентные последовательности Коши.
Доказать, что существуют два таких непересекающихся интервала I 1, I* 2, что
почти все a i лежат в I 1, а почти все b i -- в I 2.

Доказательство: Выберем такое \epsilon, для которого не найдётся отрезка
соответствующей длины, содержащего в себе почти все элементы и первой, и второй
последовательностей. Отдельно для каждой из последовательностей такие отрезки --
длины \epsilon -- найдутся, обозначим их как I 1 := [x a,y a] и I* 2 :=
[x b,y* b]. Если пересечение этих отрезков пусто, то утверждение доказано.
Предположим, что оно непусто. Длина пересечения, очевидно, меньше \epsilon,
поэтому в пересечении не могут лежать почти все члены и \{a i\}, и \{b* i\}.
Также невозможен вариант, при котором почти все члены одной последовательности
лежат внутри пересечения двух отрезков, а почти все члены другой -- снаружи,
либо в I 1, либо в I* 2. Таким образом, единственный возможный вариант -- почти
все a i лежат в I 1 \setminus (I 1 \cap I 2), а все b i лежат в I* 2 \setminus
(I 1 \cap I 2).
>> No.164425  
>>164420
Как написал бы двачерок: " define высшая математика* ".
Термин "высшая математика" сам по себе отсылает к чему-то стандартизированному. Если ты спрашиваешь "как?", значит, есть несколько стандартов и нужно либо точнее определить, какой из них имеется в виду, либо сначала нужно выбрать лучший.
Но поскольку математики не любят высшую математику, то думаю, самым уместным было бы спросить: зачем?
В конце концов, высшая математика и математика – очень разные вещи. То есть если действительно есть надобность, то можно изучать высшую математику. Если нет – можно не изучать.
>>164421
Идея правильная, лишние шаги. Если это увлечение, то полезно сделать коротко и тогда идея для тебя будет явной.
>> No.164473  
000021-9t7urkzgsh.jpg (0.0 KB, -1x-1)
0
>>160159
Добрый день! У меня возникла простейшая проблема.
Есть таблица с данными, из которых строятся простые графики. Нужно для каждого из них найти значения у при известных х. Вот только ни в Exel, ни в Statistica это не реализуется, а в R я наклепал такой говнокод, что генератор случайных чисел справляется лучше. Подскажите, пожалуйста, как это сделать, или где искать готовые решения.
>> No.164475  
--618.jpg (0.0 KB, -1x-1)
0
>>160159
Новый тред: >>164474.
>> No.164478  
>>164473
> Exel
Твоё написание этого слова огорчает Билла Гейтса (и меня).
>> No.164479  
>>164478
Пишу с холодильника, простите за неровный почерк.
>> No.164481  
--487.jpg (0.0 KB, -1x-1)
0
>>164475
Ну наконец-то.
>>164479
Прощаю.


Удалить сообщение []
Пароль